You are on page 1of 46

1.

An AIDS patient, who is being treated with multiple drugs, develops breast hypertrophy, central adiposity, hyperlipidemia, insulin resistance
and nephrolithiasis. If these changes are related to his drug treatment, this drug belongs to which group of anti-retroviral drugs?
a !ucleoside "everse #ranscriptase Inhibitors $NRTIS
b !on-nucleoside "everse #ranscriptase Inhibitors $!!"#IS
c Fusion Inhibitors
d Protease Inhibitors
e Integrase Inhibitors
1. A %& year old 'I( positive male patient comes to the )*D with complaint of anore+ia, nausea and vomiting and abdominal pain. 'is
abdomen is tender in the epigastric area. ,aboratory results reveal a raised serum amylase activity and a preliminary diagnosis is made of acute
pancreatitis. -hich of the following anti-retroviral drugs has the patient most li.ely been ta.ing?
a Sa/uinavir
b 0idovudine
c Didanosine
d 1faviren2
e 1nfuvirtide
3. A 45 year old 'I( positive patient is receiving 'AA"# regimen $'ighly active anti-retroviral therapy. 6our wee.s after initiating therapy, he
comes to the emergency department complaining of severe pain in the flan., nausea and fre/uent urination. -hich one of the following drugs is
most li.ely the cause of his symptoms?
a 0idovudine
b Indinavir
c 1faviren2
d !evirapine
e !elfinavir
%. A %5 year old man is recently diagnosed with 'I( and therapy is started. After the first wee. of therapy, the patient complains of headaches,
irritability, and nightmares. -hich one of the following anti-retroviral drugs is most li.ely to be causing these symptoms?
a 1faviren2
b Indinavir
c ,amivudine
d !evirapine
e Stavudine
4. A %& year old woman is diagnosed with chronic hepatitis 7 infection and therapy is initiated. 8ust after a few hours she comes to the
emergency department complaining of fever, chills and muscle aches. -hich one of the following drugs most li.ely caused these symptoms?
a ,amivudine
b Adefovir
c 1ntecavir
d Interferon alfa
e "ibavirin
9. A 95 year old man with .nown history of *ar.inson:s disease is to receive prophyla+is against Influen2a A virus. 'e is given a drug that is
useful against *ar.inson:s disease as well for prophyla+is against influen2a. #he drug with which of the following mechanisms of action is most
li.ely to have been given?
a It prevents entry and penetration of the virus
b It prevents uncoating of the virus
c It prevents replication of the virus
d It prevents assembly of newly synthesi2ed virus particles
e It prevents release of the newly synthesi2ed virus
;<. Antimicrobial agent of choice for the outpatient treatment of infections due to animal bite wound be=
a >efuro+ime sodium
b Amo+icillin?clavulanate potassium
c *enicillin (
d Ampicillin sodium?Salbactam sodium
e #icarcillin disodium?clavulanate potassium
;@. An antibiotic is distributed in total body water and has an elimination half life of 4& minutes. -hich one of the following statements is
consistant with that observation?
a #he drug is stored in high concentration in fat
b #he rapid rate of disappearance rules out metabolism of the drug by the liver
c #he drug is actively transported into tubular urine
d #he drug cannot be bound to plasma proteins
e #he drug is eliminated only by glomerular filtration
;A. -hich one of the following is indicated for treatment of chlamydial urethritis during pregnancy?
a Amo+icillin $Amo+il
b *enicillin (
c 1rythromycin base
d Do+ycycline $Bonodo+
e Tetracycline
;15. A 3&-year-old male has a dental infection associated with facial swelling and lymphadenopathy. -hich one of the following is the most
appropriate antibiotic?
a >ephale+in
b #etracycline
c *enicillin $note= watch an animation on penicillin mechanism of action
d 1rythromycin
e Centamicin
;11. A 19-year-old se+ually active nulliparous white female complains of pelvic pain and vaginal discharge. )n e+amination she is found to have
a temperature of %A.@D > $153.5D 6, pain with movement of the cervi+, and tenderness and a mass in the right adne+a. -hich one of the
following treatment would be appropriate?
a )utpatient treatment with penicillin C procaine ,intramuscularlyE probenecid orallyE plus do+ycycline $(ibramycin orally for 14 days and
ree+amination in % days
b )utpatient treatment with ceftria+one intramuscularlyE probenecid orallyE plus do+ycycline twice a day for 14 days and ree+amination in 1
wee.
c )utpatient treatment with cefo+itin intramuscularlyE plus do+ycycline twice a day for 14 days and ree+amination in 15 days
d 'ospitali2ation for treatment with cefo+itin intravenously and do+ycycline orally or intravenously, then do+ycycline orally twice a day to
complete 14 days of treatment
e 1rythromycin 3 grams orally as a single dose
;13. A drug that may cause nephroto+icity, is=
a *enicillin C
b 1rythrocin
c Centamycin
d >efuro+ime
e *enicillin (
;1%. -hich one of the following is a common early side effect of *enicillin?
a >onstipation
b ,oss of appetite
c )rthostatic hypotension
d Atrioventricular bloc.
e S.in rash
;14. -hich of the following might be seen in patient of #7, who has regularly been inFecting intramuscular inFection of streptomycin=
a Depression
b Sialorrhea
c Increased serum alanine aminotransferase
d *riapism
e Deafness
;1&. Significant negative interaction has been .nown to occur between the following drugs if given concurrently=
a *enicillin C GampE *enicillin (
b *enicillin GampE tetracycline
c *enicillin GampE Centamycin
d *enicillin GampE >lavulanic acid
e #icarcillin GampE >lavulanic acid
;19. -hich of the following is !)# a recogni2ed complication of ampicillin to+icity=
a 'emolytic anemia
b Diarrhea
c !ausea
d )vergrowth of gram-positive organisms
e (entricular fibrillation
;1<. Doctor is wrong when he suggests his patient to get drug=
a Streptomycin by I( route
b Centamycin by I( route
c !eomycin by oral route
d *enicillin ( by oral route
e >efepime by I( route
;1@. #elithromycin=
a Is structurally related to tetracycline
b Is structurally related to beta lactam antibiotics
c Is structurally related to aminoglycosides $note= see an animation on aminoglycosides mechanism of action
d 7inds more tightly to ribosomes and so it is a poor substrate for bacterial efflu+ pumps that mediateantibiotic resistance
e Is used for H#I
;1A. Do+ycycline is used=
a for *neumonia as drug of 1st choice
b for the prevention of #7
c for the prevention of malaria
d for the prevention of leprosy
e for the treatment of se+ually transmitted diseases
;35. >lindamycin=
a is chemically related to macrolides
b binds to %5S ribosomal subunit
c is not recommended for oral route
d cross-resistance may be observed between macrolides and >lindamycin
e is e+creted in breast mil.
;31. -hich of the following statement is true=
a nafcillin is beta-lactamase resistant antibiotic
b cefa2olin is second generation cephalosporin
c ta2obactam is used in gonorrhea mainly
d penicillin C is contraindicated in infections caused by spirochetes
e thrombocytosis is main side effect of Linezolid
;33. All of the following statement is true, e+cept=
a chloramphenicol is broad spectrum protein synthesis inhibitor
b line2olid and streptogramins are narrow spectrum protein synthesis inhibitors
c mechanism of resistance to chloramphenicol is plasmid mediated and occurs through the formation of acetyltransferases that inactivate the
drug
d tetracyclines are narrow spectrum protein synthesis inhibitors
e susceptible organisms accumulate tetracyclines intracellularly via energy dependant transport systems in their cell membranes
;3%. in a patient with culture-positive enterococcal endocarditis who has failed to respond to vancomycinbecause of resistance, the treatment
most li.ely to be effective is=
a clarithromicin
b erythromycin
c line2olid
d minocycline
e #icarcillin
;34. A 34 year old male patient is suffering from peptic ulcer. 'e was advised anti-peptic ulcer drugs including do+ycycline. -hich one of the
following statements about do+ycycline is false?
a it is bacteriostatic
b it is e+creted mainly in the feces
c it is used in ,yme disease
d it has a long elimination half-life
e it is more active than tetracycline against '. *ylori
;3&. A 13 years old female patient came to pediatrician suffering from headache, high grade fever, moderate chest pain, Foint pain and drowsy.
-hen doctor got history, he was told to ta.e dia2epam 3.& mg by patient 1& hours ago. Actually it was already a diagnosed case of pneumonia
and she was on drug treatment including streptogramin. >oncerning streptogramins, which one of the following statements is false=
a they are active against methicillin-resistant staphylococci
b they may cause a syndrome of arthralgia and myalgia
c they are used in the management of infections caused by vancomycin-resistant enterococci
d they are associated with post antibiotic effect
e they induce formation of hepatic drug-metaboli2ing en2ymes
;39. #elithromycin=
a is cell wall synthesis inhibitor
b is a .etolide structurally related to macrolides
c it binds very loosely to ribosomes so it is good substrate for bacterial efflu+ pumps that mediate resistance
d is used in ,yme disease mainly
e is used in pneumonia as drug of 1st choice
;3<. -hich statement is true about tetracyclines?
a resistance mechanisms include decreased activity of the upta.e systems and, most importantly, the development of mechanisms li.e efflu+
pumps for active e+trusion of tetracyclines
b is contraindicated in gram-positive bacterial infections
c is absolutely contraindicated in gram-negative bacterial infections
d mechanism of action of tetracycline is same as >lindamycin
e resistant organisms are .illed when tetracycline is used in large doses
;3@. A %% years old male patient has been diagnosed mycoplsma pneumoniae infection. 'e was treated by one of the broad spectrum
antibacterial drug group tetracycline 3 gram #ID for 31 days. -hat may be most appropriate to+icity?
a anemia
b hepatic necrosis
c thrombocytosis
d alopecia
e sleeplessness
;3A. an elderly debilitated patient has a fever believed to be due to an infection. 'e has e+tensive s.in lesions, scrapings of which reveal the
presence of large numbers of gram-positive cocci. #he most appropriate drug to use for treatment of this patient is=
a amo+icillin
b salbactam
c cefo+itin
d nafcillin
e penicillin C
;%5. A 31 year old man was seen in a clinic with a complaint of dysuria and urethral discharge of yellow pus. 'e has a painless clean-base ulcer
on the penis and nontender enlargement of the regional lymph nodes. Cram stain of the urethral e+udates showed gram negative diplococci
within polymorphonucleocytes. #he patient informed the clinic staff that he was unemployed and had not eaten a meal for two days. #he most
appropriate treatment of gonorrhea in this patient is=
a amo+icillin orally for < days
b vancomycin intramuscularly as a single dose
c ceftria+one intramuscularly as a single dose
d tetracycline orally for < days
e procaine penicillin C intramuscularly as a single dose plus one gram of probenecid
;%1. 1ighty years old male patient having <% Ig body weight was suffering from fever, headache and lumbar pain since last < days. 'e was
e+amined in clinic. Cram stain of the smear of >S6 revealed gram positive rods resembling diphtheroids. #he antibiotic regimen for empiric
treatment would include=
a 1rythromycin
b #icarcillin
c >efotetan
d >efa2olin
e Ampicillin
;%3. A 3& year old male presents to the medical )*D with complaint of cough and low grade fever since the last % months. ,aboratory tests
show sputum positive for acid fast bacteria. -hich of the following combination of drugs is most li.ely to be administered at the start to this
patient?
a Streptomycin, Isonia2id, "ifampicin and *yra2inamide
b *AS, *yra2inamide and "ifampicin
c *yra2inamide, 1thambutol and Isonia2id
d Streptomycin and "ifampicin
e Streptomycin alone
;%%. A 3& year old male has been given treatment for pulmonary tuberculosis for 3 months as initial phase. !ow he comes to the )*D for
medication for the continuation phase. -hich of the following drugs will he most li.ely be given for the continuation phase?
a *yra2inamide and Isonia2id
b 1thambutol and Streptomycin
c Isonia2id and "ifampicin
d "ifampicin and Streptomycin
e 1thambutol and *yra2inamide
;%4. A young man 39 years old is started treated for pulmonary tuberculosis. 'e comes to the medical )*D after 1 wee. with complaint of
orange red colored urine and sweat and tears. -hich of the following drugs is most li.ely causing these symptoms?
a Streptomycin
b "ifampicin
c 1thambutol
d *yra2inamide
e #hiaceta2one
;%&. A %5 year old male with .nown #7 infection and ta.ing medication since 1 month comes to the 1ye )*D with complaint of visual
disturbances including decreased visibility and slight color blindness. If these symptoms are due to his drug treatment, which one of the
following drugs is most li.ely the cause of his symptoms?
a Streptomycin
b >iproflo+acin
c 1thambutol
d "ifampicin
e Isonia2id
;%9. A %& year old male recently diagnosed with tuberculosis and is started on medication. After a few days he comes to the emergency
department a few hours after ta.ing a meal at BcDonald:s with severe pain in the big toe. 7lood tests show a very high uric acid level. -hich of
the following drugs most li.ely caused these symptoms?
a Streptomycin
b "ifampicin
c Isonia2id
d Ami.acin
e *yra2inamide
;%<. A 45 year old male ta.ing medication for tuberculosis since the last 3 months comes to the 1!# )*D with complaint of hearing problems
and ringing in the years. #he drug with which of the following mechanisms of action is most li.ely causing these symptoms?
a 7inds to %5 S "ibosome subunit GampE inhibits initiation comple+
b Inhibits D!A Dependent "!A *olymerase
c Inhibits synthesis of mycolic acid
d Inhibits synthesis of arabinoglycan subunits
e Inhibits D!A Cyrase
;%@. A %& year old female ta.ing medication for tuberculosis and oral contraceptives comes to the Cynaecology )*D with complaint of
amenorrhea. ,aboratory test shows a positive pregnancy test. -hich one of the following drugs most li.ely caused her to become pregnant?
a "ifampicin
b *yra2inamide
c Streptomycin
d 1thambutol
e Isonia2id
;%A. A 4& year old male with diagnosed chronic liver disease gets infected with the tuberculosis. -hich one of the following drugs would be
most safe for this patient for the treatment of tuberculosis?
a Isonia2id
b "ifampin
c *yra2inamide
d 1thambutol
e 1thionamide
;45. A 4@ year old male is diagnosed with tuberculosis and has been treated for 3 months with isonia2id and rifampicin with good response. 7ut
the patient develops numbness and paresthesias in the e+tremities. -hat will you most li.ely do ne+t?
a Add pyrido+ine
b Stop isonia2id
c Add vitamin 1
d "eplace rifampin with ethambutol
e "eplace isonia2id with streptomycin
;41. A %5 year old male presents with history of fever occurring in a step-ladder fashion for the last 15 days. 'e also feels abdominal pain after
ta.ing meals. ,ab tests show decreased #,> count and a positive -idal test. #he drug which is most li.ely to be given acts by which one of the
following mechanisms?
a >iproflo+acin
b Ampicillin
c >otrimo+a2ole
d >efi+ime
e A2ithromycin
;43. A %& year old pregnant woman presents to the gynecology department with complaint of pain with rising fever since the last & days. ,ab
tests show gram negative bacilli and widal test comes out positive. -hich one of the following drugs will most li.ely be administered?
a >iproflo+acin
b ,evoflo+acin
c )flo+acin
d !orflo+acin
e Ampicillin
;4%. A 3< year old male football player gets inFured in a match. 'e comes to the emergency department with pain and swelling near the an.le.
)n e+amination, there is small wound with a tender swelling near the an.le Foint. 6urther tests reveal tendon rupture with tendonitis setting in.
-hich one of the following antibiotics, if re/uired, will most probably not be given to this patient?
a >eftria+one
b >larithromycin
c Aminoglycoside
d >iproflo+acin
e Amo+icillin
;44. A 45 year old male comes to the medical )*D with symptoms of cough GampE fever for the last few days. Sputum culture shows presence
of gram positive cocci. -hich one of the following 6luoro/uinolones is most li.ely to be administered to this patient?
a >iproflo+acin
b !orflo+acin
c )flo+acin
d Catiflo+acin
e ,evoflo+acin
;4&. A &5 year old patient comes to the emergency department, a day after he has been administered an antibiotic for an on-going infection,
with complaint of palpitations. 1>C reveals prolonged ;# interval. -hich one of the following 6luoro/uinolones most li.ely caused this adverse
effect?
a >iproflo+acin
b !orflo+acin
c ,evoflo+acin
d )flo+acin
e Catiflo+acin
;49. Disinfectants are=
a Strong chemical agents that inhibit or .ill micro-organisms
b Agents with sufficient to+icity for host cells
c #hey .ill both vegetative cells and spores
d A process intended to .ill or remove all types of micro-organisms including spores and include viruses
e A process that .ills non-sporulating microorganisms by hot water or steam at 9&-155 >
;4<. A patient with watery stools is diagnosed as suffering from amebic dysentery. 'e is given a drug that causes a metalic taste in the mouth
which drug may be given.
a Iodo/uinol
b Dilo+anide furate
c Betronida2ole
d *entamiline
e 1metine.
;4@. #he reason for giving Betronida2ole for oropharyngeal infection is due to its good activity against=
aCram positive cocci
bCram !egative cocci
cCram positive bacilli
dCram negative bacilli
eAnaerobes li.e 7. fragilis
;4A. *atient comes to emergency with the complaints of mar.ed visual and auditory abnormities, vomiting diarrhea abdominal pain GampE s.in
rashes. 6rom the history of the patient it was revealed that he too. some drug for the treatment of fever. -hat could be probable diagnosis?
a>inchonism
bAcute gastro enteritis
c>holera
d#yphoid fever
eAcid peptic disease
;&5. A %< year old male is having repeated episodes of fever with chills with blood smear positive for malaria since last one year. 'e was given
chloro/uine each time GampE the fever subsided. -hich of the following drugs would you add with chloro/uine this time?
aArtemether
bBeflo/uine
cBalarone
d*rima/uine
e'alofantrine
1 J Introduction to *harmacology= 7asic *rinciples
1 -hich of the following is !)# part of the etymology of the word pharmacology?
a Bedicine
b Drug
c 'erb
d *oison
e Study
3.1 -hich of the following describes an agonist?
a Any substance that brings about a change in biologic function through its
chemical action
b A specific regulatory molecule in the biologic system where a drug interacts
c A drug that binds to a receptor and stimulates cellular activity
d A drug that binds to a receptor and inhibits or opposes cellular activity
e A drug directed at parasites infecting the patient
3.3 Kenobiotics are considered=
a 1ndogenous
b 1+ogenous
c Inorganic poisons
d #o+ins
e ,igands
3.% -hich of the following would be a to+in $poison of biological origin?
a *b
b As
c 'g
d Atropine
3.4 #he vast maFority of drugs have molecular weights $B- between 155 and 1,555.
,arge drugs, such as alteplase $t-*A, must be administered=
a Into the compartment where they have their effect
b )rally so they do not absorb too /uic.ly
c "ectally to prevent irritation to the stomach lining and vessels
d (ia the intraosseous $I) route
e #itrated with buffering agents to prevents cell lysis
3.& -hich of the following occurs with drugs that are e+tremely small, such as Lithium?
a "eceptor mediated endocytosis
b Binor drug movement within the body
c (asodilation when inFected intravenously $I(
d Specific receptor binding
e !onspecific binding
3.9 Drugs fit receptors using the loc. and .ey model. >ovalent bonds are the LLLL and
the LLLL specific.
a StrongestE Bost
b StrongestE ,east
c -ea.estE Bost
d -ea.estE ,east
3.< -arfarin $>oumadin is given as a racemic mi+ture with the S enantiomer being four
times more active than the " enantiomer. If the mi+ture of -arfarin given is &5M S and
&5M ", what is the potency compared with a 155M " enantiomer solution?
a 4 N " O 1 N S P 1
b 4 N " O 1 N S P 1.&
c 4 N " O 1 N S P 3
d 4 N " O 1 N S P 3.&
e 4 N " O 1 N S P 4
3.@ -hat determines the degree of movement of a drug between body compartments?
a *artition constant
b Degree of ioni2ation
c p'
d Si2e
e All of the above
%.1 -hich of the following is !)# a protein target for drug binding?
a Side of action $transport
b 1n2ymes
c >arrier molecules
d "eceptors
e Ion channels
%.3 -hich of the following is an e+ample of a drug acting directly through receptors?
a *rotamine binds stoichiometrically to heparin anticoagulants
b Adrenergic beta bloc.ers for thyroid hormone-induced tachycardia
c Epinephrine for increasing heart rate and blood pressure
d >ancer chemotherapeutic agents
e Bannitol for subarachnoid hemmorhage
4.1 -hat is added with drug subclassification, such as an antitubercular drug versus an
antibacterial drug?
a >ost
b Si2e
c Ioni2ation
d *recision
e Speed
4.3 -hat type of drug is propranolol $Inderal?
a nticonvulsive
b Antihypertensive
c Antinauseant
d Antihistamine
e Antipyretic
&.1 -hich of the following is considered the brand name?
a *ropranolol
b Inderal
c Adrenergic Q-bloc.er
d Roff labelS use
e 7loc.s Q-receptors in heart myocardium
&.3 -hich of the following is considered the class?
a *ropranolol
b Inderal
c Adrenergic Q-bloc.er
d Roff labelS use
e 7loc.s Q-receptors in heart myocardium
&.% -hich of the following cases would be contraindicated for propranolol $Inderal?
a 'ypertension
b 1ssential tremor
c Angina
d #achycardia
e Asthma
&.4 -hich of the following adverse effects $side-effects is !)# commonly seen with
cholinergic antagonists?
a 7lurred vision
b >onfusion
c Biosis
d >onstipation
e Hrinary retention
9.1 #he drug chloramphenicol $>hloromycetin is ris.y for which of the following?
a !eonates
b Ceriatric patients
c Adult males
d )bese patients
e >ongestive heart failure patients
9.3 'ow does the glomerular filtration rate $C6" change after the age of 45?
a Increase 1M each year
b Increases 3M each year
c Decreases 1M each year
d Decreases 3M each year
e Does not depend on age
9.% A decrease in renal and liver function, as seen in the elderly, would prolong drug
half-life, LLLL plasma protein binding, and LLLL volume of distribution.
a IncreaseE Increase
b DecreaseE Decrease
c IncreaseE Decrease
d DecreaseE Increase
9.4 -hen prescribing isonia2id $"imifon, pharmacogenetics must be considered as
TA5M of Asians and certain other groups are LLLL acetylators, and thus have a LLLL
blood concentration of a given dose and a decreased ris. of to+icity.
a SlowE Increased
b SlowE Decreased
c 6astE Increased
d 6astE Decrease
9.& -hich of the following are the two modifying factors that contribute to why women
have higher blood pea. concentrations of alcohol than men when consuming e/uivalent
amounts?
a ,ower blood volume G increased hormones
b ,ower fat content G more gastric alcohol dehydrogenase $AD'
c 'igher fat content G more gastric alcohol dehydrogenase $AD'
d ,ower fat content G less gastric alcohol dehydrogenase $AD'
e 'igher fat content G less gastric alcohol dehydrogenase $AD'
3 J *harmaco.inetic *rinciples= Drug Bovement
1 *harmaco.inetics is the effect of the LLLL and pharmacodynamics is the effect of the LLLL.
a Drug on a drugE 7ody on the drug
b 7ody on the drugE Drug on a drug
c Drug on the bodyE 7ody on the drug
d 7ody on the drugE Drug on the body
e Drug on a drugE Drug on a drug
3.1 -hich of the following is !)# an action of the body on a drug?
a Absorption
b Distribution
c Betabolism
d 1+cretion
e Side effects
% If a drug is @5M bound to blood elements or plasma proteins, what part is considered the free form?
a 35M
b 45M
c &5M
d @5M
e 155M
4.1 -hich of the following describes minimal effective concentration $B1>?
a #he minimal drug plasma concentration that can be detected
b #he minimal drug plasma concentration to enter tissues
c #he minimal drug plasma concentration to interact with receptors
d #he minimal drug plasma concentration to produce effect
e #he minimal drug plasma concentration to reach therapeutic levels
4.3 If a patient misses three doses of their daily drug, which of the following $in general is the best solution?
a #a.e a 4+ dose at the ne+t dose time
b -ait % more days $wee. total then return to normal regimen
c Do nothing and continue normal regimen
d Setup an appointment to have the patient evaluated
e *rescribe a higher dosage pill so missed doses will have less effect
4.% 7lood levels of a drug correlate to the effectiveness of that drug, such as with
penta2ocine $#alwin or phenobarbitol $,uminal.
a #rue
b 6alse
&.1 -hich of the following drug permeation mechanisms involves polar substances too large to enter cells by other means, such as iron or
vitamin 713?
a A/ueous diffusion
b ,ipid diffusion
c >arrier molecules
d 1ndocytosis and e+ocytosis
&.3 -hich of the following drug permeation mechanisms occurs across epithelial tight
Functions and is driven by a concentration gradient?
a A/ueous diffusion
b ,ipid diffusion
c >arrier molecules
d 1ndocytosis and e+ocytosis
&.% -hich of the following drug permeation mechanisms uses the 'enderson-
'asselbalch e/uation for the ratio of solubility for the wea. acid or wea. base?
a A/ueous diffusion
b ,ipid diffusion
c >arrier molecules
d 1ndocytosis and e+ocytosis
&.4 -hich of the following drug permeation mechanisms is used for peptides, amino
acids, glucose, and other large or insoluble molecules?
a A/ueous diffusion
b ,ipid diffusion
c >arrier molecules
d 1ndocytosis and e+ocytosis
&.& -hich of the following drug permeation mechanisms uses caveolae?
a A/ueous diffusion
b ,ipid diffusion
c >arrier molecules
d 1ndocytosis and e+ocytosis
9.1 Hsing the 6ic. ,aw of Diffusion, how will flu+ change if membrane thic.ness is
doubled?
a It will double
b It will /uadruple
c It will halve
d It will /uarter
e It will not change
9.3 Hsing the 6ic. ,aw of Diffusion, how will flu+ change if the permeability
coefficient is /uadrupled?
a It will double
b It will /uadruple
c It will halve
d It will /uarter
e It will not change
<.1 -hich of the following is the amount of a drug absorbed per the amount
administered?
a !ioavailability
b !ioe"uivalence
c Drug absorption
d 7ioine/uivalence
e Dosage
<.3 -hich of the following is !)# needed for drug bioe/uivalence?
a Same active ingredients
b Same strength or concentration
c Same dosage form
d Same route of administration
e Same side effects
<.% 6or intravenous $I( dosages, what is the bioavailability assumed to be?
a 5M
b 3&M
c &5M
d <&M
e 155M
<.4 Although morphine $Avin2a, )ramorph S", BS >ontin is well-absorbed when
administered orally #P$%, how much of the drug is metaboli2ed on its first pass through
the liver?
a A5M
b <5M
c &5M
d %5M
e 15M
<.& 6or a generic drug to be bioe/uivalent to an innovator drug $per 6DA, it must be
measured in LLLL of subFects to fall within LLLL of the mean of the test population
bioavailability.
a &5E &5
b @5E 35
c 35E @5
d A&E &
e &E A&
<.9 Hsing the 6DA bioe/uivalence rule, how much variation could a generic drug
potentially have from an innovator and still be considered e/uivalent?
a 155M
b 35M
c 45M
d 95M
e @5M
@.1 -hich of the following is !)# a pharmaco.inetic process?
a Alteration of the drug by liver en2ymes
b Drug metabolites are removed in the urine
c Bovement of drug from the gut into general circulation
d #he drug causes dilation of coronary vessels
e #he drug is readily deposited in fat tissue
@.3 -hich of the following can produce a therapeutic response? A drug that is=
a 7ound to plasma albumin
b >oncentrated in the bile
c >oncentrated in the urine
d !ot absorbed from the CI tract
e Hnbound to plasma proteins
@.% -hich of the following most correctly describes steroid hormones with respect to
their ability to gain access to intracellular binding sites?
a #hey cross the cell membrane via a/ueous pores
b #hey have a high permeability coefficient
c #hey are passively transported via membrane carriers
d #hey re/uire vesicular transport
e #heir transport re/uires the hydrolysis of A#*
% J *harmaco.inetic *rinciples= p' and Drug Bovement
1 Bost drugs are either LLLL acids or LLLL bases.
a StrongE Strong
b StrongE -ea.
c -ea.E -ea.
d -ea.E Strong
3.1 Aspirin readily donates a proton in a/ueous solutions and pyrimethamine readily
accepts a proton in a/ueous solution. #hus, aspirin is a$b LLLL and pyrimethamine is
a$n LLLL.
a AcidE 7ase
b 7aseE Acid
c AcidE Acid
d 7aseE 7ase
3.3 Civen the e/uilibrium 'A UPT A- O 'O $acid and 7'O UPT 7 O 'O $base, in an
acid environment $low p' the acid reaction will move to the LLLL and the base reaction
will move to the LLLL.
a "ightE ,eft
b "ightE "ight
c ,eftE "ight
d ,eftE ,eft
%.1 -hat form of a drug is more lipid-soluble, and thus would remain trapped within a
compartment where the p' does not favor the lipid-soluble form?
a Strong acid $A-
b -ea. acid $A-
c !eutral $A' and 7
d -ea. base $7'O
e Strong base $7'O
%.3 #he lipid-soluble form of a base is LLLL and the lipid-soluble form of an acid is
LLLL.
a *rotonatedE *rotonated
b *rotonatedE Hnprotonated
c HnprotonatedE Hnprotonated
d HnprotonatedE *rotonated *harmacology J *art 1 ;ui2
(ersion= 19)ct355@ *age @ of 43
4.1 If the pIa of Aspirin $acetylsalicylic acid is %.& and the p' of the stomach is 3.&,
how much Aspirin is in the protonated species in the stomach and is this the amount
available for absorption?
a V A1ME Wes
b V A1ME !o
c V AME Wes
d V AME !o
4.3 -hat percentage of Aspirin would be ioni2ed in the blood compartment $p' P <.4
assuming p' is <.& and Aspirin pIa is %.&?
a $15,555 J 1 ? 1 P AA.AAM
b $155 J 1 ? 1 P AAM
c !one
d 1 ? $155 J 1 P 5.AM
e 1 ? $15,555 J 1 P 5.55AM
4.% If the p' J pIa P -1, what percentage of wea. base is nonioni2ed?
a AA
b A5
c &5
d 15
e 1
4.4 If the p' J p.a P 3, what percentage of wea. acid is nonioni2ed?
a AA
b A5
c &5
d 15
e 1
4.& If p' T pIa, the drug is LLLL and if p' U pIa, the drug is LLLL. An unprotonated
acid is LLLL and a protonated base is LLLL.
a *rotonatedE HnprotonatedE >hargedE >harged
b *rotonatedE HnprotonatedE !eutralE !eutral
c HnprotonatedE *rotonatedE >hargedE >harged
d HnprotonatedE *rotonatedE !eutralE >harged
e HnprotonatedE *rotonatedE >hargedE !eutral
&.1 -ea. acids are e+creted faster in LLLL urine and wea. bases are e+creted faster in
LLLL urine.
a AcidicE Al.aline
b Al.alineE Acidic
c AcidicE !eutral
d !eutralE Al.aline
e Al.alineE !eutral
&.3 A patient presents with an overdose of acidic Aspirin. #he drug LLLL can be given to
LLLL the p' of the urine and trap the Aspirin, preventing further metabolism.
a !a'>)%E Increase
b !a'>)%E Decrease
c !'4>lE Increase
d !'4>lE Decrease *harmacology J *art 1 ;ui2
(ersion= 19)ct355@ *age A of 43
&.% A patient presents with an overdose of al.aline >odeine. #he drug LLLL can be given
to LLLL the p' of the urine and trap the >odeine, preventing further metabolism.
a !a'>)%E Increase
b !a'>)%E Decrease
c !'4>lE Increase
d !'4>lE Decrease
9.1 #he principle of drug manipulation for e+cretion of a drug out of the renal tubule can
be accomplished by=
a Acidifying the urinary p'
b AdFusting the urinary p' to protonate wea.ly acidic drugs
c AdFusting the urinary p' to unprotonate wea.ly basic drugs
d AdFusting the urinary p' to ioni2e the drug
e 7y neutrali2ing the urinary p'
9.3 Aspirin is a wea. organic acid with a pIa of %.&. -hat percentage of a given dose
will be in the lipid-soluble form at a stomach p' of 1.&?
a About 1M
b About 15M
c About &5M
d About A5M
e About AAM
9.% 6or which of the following drugs is e+cretion most significantly accelerated by
acidification of the urine?
a -ea. acid with pIa of &.&
b -ea. acid with pIa of %.&
c -ea. base with pIa of <.&
d -ea. base with pIa of <.1
9.4 A patient diagnosed with type 3 diabetes is administered an oral dose of 5.1 mg
chloropropamide, an insulin secretagogue and wea. acid with a pIa of &.5. -hat is the
amount of this drug that could be absorbed from the stomach at p' 3.5?
a AA.A Xg
b A5 Xg
c &5 Xg
d 5.5& mg
e 5.51 mg
4 J *harmaco.inetic *rinciples= Absorption
1 7ioavailability $6 is the fraction or percentage of administered drug that reaches the
systemic circulation via a given route as compared to what route?
a )ral
b I( $intravenous
c I) $intraosseous
d >S6 $cerebrospinal fluid
e -hatever route attains the target drug concentration in plasma $>#
3 -hat organ is responsible for metabolism in the Rfirst pass effectS?
a 7rain
b 'eart
c Iidney
d ,iver
e Spleen
%.1 A patient is in the hospital and is stable on digo&in 5.1<& mg I( /d $daily. 'ow
much digo+in in mg. would you need to give your patient orally, given that the
bioavailability for oral digo+in tablets is 5.<?
a $5.1<& N 5.< ? $1.5 P 5.133& mg
b $5.1<& N 1 ? $5.< P 5.3& mg
c $5.1<& O 5.< ? $1.5 P 5.@<& mg
d $5.1<& O 1 ? $5.< P 1.9< mg
e !o change is necessary
%.3 Civen a graph of plasma drug concentration versus time, what part of the graph
would be used to calculate bioavailability for a *) $oral drug administration?
a Ba+imum concentration
b Steady concentration
c Derivative of the curve $slope
d Integral of the curve $area underneath
e #he curve is not used to calculate bioavailability
4.1 -hich of the following routes of administration has a bioavailability of about @5-
155M, is usually very slow absorbing, and has prolonged duration of action?
a I( $intravenous
b IB $intramuscular
c S; $subcutaneous
d "ectal
e #ransdermal
4.3 -hich of the following routers of administration is the most convenient, although
may have a bioavailability anywhere from &-155M?
a *) $oral
b I( $intravenous
c IB $intramuscular
d S; $subcutaneous
e #ransdermal
4.% -hich of the following enteral administration routes has the largest first-pass effect?
a S, $sublingual
b 7uccal
c "ectal
d )ral
4.4 1pithelial cells are connected by LLLL, which are tough to cross and materials often
must pass through the cells. 1ndothelial cells of blood vessels are connected by LLLL,
which proteins cannot cross but smaller drugs $B- 355-&55 can.
a Bacular gap FunctionsE #ight Functions
b #ight FunctionsE Bacular gap Functions
c Adherens FunctionsE #ight Functions
d #ight FunctionsE Adherens Functions
e Bacular gap FunctionsE Adherens Functions
4.& -hich of the following administration routes is not often used, is painful, and has a
ris. of infection and adhesion?
a 1*I $epidural
b IA $intraarterial
c I* $intraperitoneal
d I( $intravenous
e I) $intraosseous
4.9 -hich of the following is !)# an advantage of prolonged release medications?
a ,ess fre/uent administration
b #herapeutic effect overnight
c ,ower incidence of side effects
d *atient compliance
e Bore fluctuation in plasma concentration
4.< -hat is the common location for the scopolamine motion sic.ness transdermal
patch?
a Side of the hip
b >hest
c )ver the deltoid muscle
d 7ehind the ear
e )n the bac. of the nec.
& J *harmaco.inetic Distribution= 7asics
1.1 -hich of the following would receive drug slowly?
a ,iver
b 7rain
c 6at
d Buscle
e Iidney
1.3 -hich of the following is the least important for passage through capillary walls but
the most important for passage through the cell wall?
a Bolecular si2e
b ,ipid solubility
c Diffusion constant
d p'
e pIa
1.% -hich of the following is the most important for movement through capillary walls?
a Bolecular si2e
b ,ipid solubility
c Diffusion constant
d p'
e pIa
1.4 -hich of the following locations would most trap a lipid soluble drug?
a 7lood
b Intestines
c 7rain
d Stomach
1.& -hat type of drugs can cross the blood-brain barrier $777?
a ,arge and lipid-soluble
b ,arge and lipid-insoluble
c Small and lipid-soluble
d Small and lipid-insoluble
3.1 Acidic drugs, such as phenytoin, bind primarily to which of the following plasma
proteins?
a Y1-fetoprotein $A6*
b C> Clobulin
c Albumin
d Y1-acid glycoprotein $AAC
e #ranscortin
3.3 7asic drugs, such as lidocaine, bind primarily to which of the following plasma
proteins?
a Y1-fetoprotein $A6*
b Cc-Clobulin $CcC
c Albumin
d Y1-acid glycoprotein $AAC
e #ranscortin
%.1 A decrease in drug-protein binding will lead to which of the following?
a Decrease in the unbound drug concentration
b Increase in free drug
c Increase in rate of drug elimination
d Decrease in volume of distribution
%.3 A patient presents with acute-onset cirrhosis of the liver. #hey are found to have
hypoalbuminemia. In severe cirrhosis it is e+pected that AAC will be decreased, but the
patient presents with increased AAC due to the inflammatory response. -hich of the
following is the most li.ely?
a Increased acidic drug binding and increased basic drug binding
b Increased acidic drug binding and decreased basic drug binding
c Decreased acidic drug binding and increased basic drug binding
d Decreased acidic drug binding and decreased basic drug binding
%.% -hich of the following is !)# a site of loss $where drug is not used?
a 6at
b CI tract
c Buscle
d Site lac.ing receptors
4.1 -hich of the following locations can accumulate lipid-soluble drugs, has little or no
receptors, and can hold distributed drugs li.e barbiturates?
a ,iver
b Iidney
c 7rain
d 6at
e 6etus
4.3 -hich of the following locations has high blood flow and is a site of e+cretion?
a ,iver
b Iidney
c 7rain
d 6at
e 6etus
4.% Anything affecting renal perfusion will affect drug delivery to the .idney, drug
e+cretion, and drug levels in the blood.
a #rue
b 6alse
4.4 -hich of the following can be treated with drugs due to a lea.y area in the blood-
brain barrier near the medulla?
a Sei2ures
b Shivers
c Diarrhea
d !ausea
e (omitting
4.& -hat is the appro+imate lag time for e/uilibration between maternal blood and fetal
tissues?
a 35 mins
b 45 mins
c 1 hour
d 3 hours
e 9 hours
Batch the body compartment with the volume, assuming a <5.g male patient=
&.1 #otal body a 4
&.3 *lasma b 15
&.% Interstitial c 14
&.4 1+tracellular d 3@
&.& Intracellular e 43
&.9 If protein plasma binding is decreased, how will volume of distribution be affected?
a Increased
b Decreased
c !ot changed
&.< 455 mg of a drug is administered to a patient and the drug is later measured in
plasma to be 1 Xg?ml. -hat is the apparent volume of distribution #'d%?
a 5.54 ,
b 5.4 ,
c 4 ,
d 45 ,
e 455 ,
&.@ 1lderly patients often have LLLL muscle mass and thus a$n LLLL (d.
a BoreE Increased
b BoreE Decreased
c ,essE Increased
d ,essE Decreased
&.A *atients with ascites or edema would have LLLL (d for hydrophilic drugs, such as
gentamicin.
a Increased
b Decreased
c Hnchanged
9 J *harmaco.inetics= Drug Betabolism
1.1 -hich of the following locations is the most li.ely for finding a free, unaltered drug?
a Hrine
b 6eces
c 7reast mil.
d 6at
e Sweat
1.3 Bost drugs are active in their LLLL form and inactive in their LLLL form.
a !on-polarE *olar
b *olarE !on-polar
c -ater-solubleE ,ipid-soluble
d ,ipid-insolubleE -ater-insoluble
e !eutralE !eutral
3.1 Drug biotransformation phase I ma.es drugs LLLL polar for metabolism and phase II
ma.es drugs LLLL polar for e+cretion.
a BoreE Bore
b BoreE ,ess
c ,essE Bore
d ,essE ,ess
3.3 -hich of the following is !)# a phase II substrate?
a Clucuronic acid
b Sulfuric acid
c Acetic acid
d Amino acids
e Alcohol
% -hich of the following reactions is phase II and !)# phase I?
a )+idations
b "eductions
c >onFugations
d Deaminations
e 'ydrolyses
4 -hich of the following metabolically active tissues is the principle organ for drug
metabolism?
a S.in
b Iidneys
c ,ungs
d ,iver
e CI #ract
&.1 Damage at which of the following locations would most affect the goals of phase II
biotransformation?
a S.in
b Iidneys
c ,ungs
d ,iver
e CI #ract
Batch the biotransformation reaction with the drug=
&.3 'ydro+ylation of aromatic ring to increase polarity a >odeine
&.% !-deal.ylation b Borphine
&.4 Sulfo+idation c #hiorida2ine
&.& )-deal.ylation d !icotine
&.9 !-o+idation e *henobarbitol
&.< Side chain o+idation with -)' to increase polarity f *entobarbitol
&.@ >onversion to glutathione and reactive intermediate g Acetaminophen
9.1 -hat is the goal of the *4&5 system $microsomes pinched off from endoplasmic
reticulum?
a Betabolism of substances
b Deto+ification of substances
c Increasing p' of compartments containing substances
d Decreasing p' of compartments containing substances
e A G 7
9.3 "egarding the microsomal drug metaboli2ing system, a patient with late stage
alcoholism and liver damage would have more 1#)' available due to which of the
following concepts?
a Increased induction
b Decreased induction
c Increased inhibition
d Decreased inhibition
9.% "egarding the microsomal drug metaboli2ing system, a patient who is a chronic user
of barbiturates would need more drug to produce the same effects due to which of the
following concepts?
a Increased induction
b Decreased induction
c Increased inhibition
d Decreased inhibition
9.4 -hich of the following are the drugs that induce >W* 1A3 and the drugs that have
their metabolism induced by 1A3?
a >arbama2epine G phenobarbitolE #heophyline G warfarin
b *henobarbitol G phenytoin E *henytoin G warfarin
c >arbama2epine G phenytoinE -arfarin
d >arbama2epineE >yclosporine
9.& -hich of the following are the drugs that inhibit >W* 1A3 and the drugs that have
their metabolism inhibited by 1A3?
a SS"IsE *henytoin G warfarin
b Amiodarone G cimetidineE *henytoin G warfarin
c >imetidine, erythromycin, G grapefruit FuiceE #heophyline G warfarin
d >imetidine G erythromycinE >yclosporine
9.9 -hich of the following groups of people is the least li.ely to have biotransformation
effects due to altered hepatic function?
a Infants
b Adults
c 1lderly
d >hronic alcoholics
e Acetaminophen overdoses
9.< In what location does amino acid conFugation of glycine $e.g. salicyclic acid ta.e
place?
a Bicrosomal
b >ytosol
c Bitochondria
9.@ -here does acetylation conFugation $e.g. isonia2id and sulfate conFugation $e.g.
acetaminophen ta.e place?
a Bicrosomal
b >ytosol
c Bitochondria
9.A -here does glucuronide conFucation $e.g. digo+in, bilirubin ta.e place?
a Bicrosomal
b >ytosol
c Bitochondria
9.15 -hat is a result of conFugation of isonia2id via !-acetylation?
a Deto+ification of liver
b Deto+ification of .idneys
c Deto+ification of blood
d Deto+ification of urine
e 'epatoto+icity
< J *harmaco.inetics= *rinciples of 1liminations
1.1 )ne liter contains 1,555 mg of a drug. After one hour, A55 mg of the drug remains.
-hat is the clearance?
a 155 m,
b 155 m,?hr
c 1 mg?ml
d 155 mg
e 1 mg?sec
1.3 #o maintain a drug concentration at steady state, the dosing rate should e/ual the
elimination rate. -hich of the following is true? $>, P Drug >learance
a Dosing rate P >, O target concentration
b Dosing rate P >, J target concentration
c Dosing rate P >, N target concentration
d Dosing rate P >, ? target concentration
1.% -hich of the following is most useful in determining the rate of elimination of a
drug, in general?
a Drug concentration in urine $renal elimination
b Drug concentration in stool $bilary elimination
c Drug concentration in blood
d Drug concentration in brain
e Drug o+idation rate
3.1 6or first-order drug elimination, half life t$1?3 is LLLL at two places on the curve
and a constant LLLL is lost per unit time.
a 1/ualE Amount
b 1/ualE *ercentage
c !ot e/ualE Amount
d !ot e/ualE *ercentage
3.3 6or first-order drug elimination, given the half-life e/uation of t$1?3 P $5.9A% N (d
? >,, how many half-lives would be necessary to reach steady state $VA&M without a
loading dose?
a 1 to 3
b 3 to %
c % to 4
d 4 to &
e & to 9
3.% -hich of the following is !)# a drug e+hibiting 2ero-order elimination .inetics?
a Aspirin
b Borphine
c *henytoin
d 1#)'
3.4 6or 2ero-order drug elimination, half-life t$1?3 is LLLL at two places on the curve
and a constant LLLL is lost per unit time.
a 1/ualE Amount
b 1/ualE *ercentage
c !ot e/ualE Amount
d !ot e/ualE *ercentage
3.& If a drug with a 3-hour half life is given with an initial dose of @ mcg?ml, assuming
first-order .inetics, how much drug will be left at 9 hours?
a @ mcg?ml
b 4 mcg?ml
c 3 mcg?ml
d 1 mcg?ml
e 5.& mcg?ml
%.1 -hat are the units for steady-state concentration $>ss, or infusion rate over
clearance?
a mg?min
b ml?min
c mg?ml
d ml?mg
e min?mg
%.3 -hat percentage of the steady-state drug concentration is achieved at %.% N t$1?3?
a 15M
b 3&M
c &5M
d <&M
e A5M
4.1 Increasing the rate of infusion changes the time necessary to reach the steady-state
concentration.
a #rue
b 6alse
4.3 An inFection of two units of a drug once-daily $/d will yield the same steady-state
concentration as an inFection of one unit of a drug twice-daily $bid.
a #rue
b 6alse
&.1 -hich of the following drugs would most li.ely need a loading dose to help reach
therapeutic levels?
a Acetaminophen, t$1?3 P 3 h
b Aspirin, t$1?3 P 1& m
c #etracycline, t$1?3 P 11 h
d Digito+in, t$1?3 P 191 h
e Adenosine, t$1?3 P 15 s
&.3 A target concentration of <.& mg?, of theophylline is re/uired for a 95 .g patient.
-hat is the loading dose, given the following= (d P 5.& ,?.g, >l P 5.54 ,?.g?hr, t$1?3 P
A.% hr?
a 5.& ,?.g N 95 .g N <.& mg?, P 33& mg?h, infusion
b 5.& ,?.g N 95 .g N <.& mg?, P 33& mg, bolus
c 5.54 ,?.g?hr N 95 .g N <.& mg?, P 1@ mg?h, infusion
d 5.54 ,?.g?hr N 95 .g N <.& mg?, P 1@ mg, bolus
&.% A target concentration of <.& mg?, of theophylline is re/uired for a 95 .g patient.
-hat is the steady state maintenance dose, given the following= (d P 5.& ,?.g, >l P 5.54
,?.g?hr, t$1?3 P A.% hr?
a 5.& ,?.g N 95 .g N <.& mg?, P 33& mg?h, infusion
b 5.& ,?.g N 95 .g N <.& mg?, P 33& mg, bolus
c 5.54 ,?.g?hr N 95 .g N <.& mg?, P 1@ mg?h, infusion
d 5.54 ,?.g?hr N 95 .g N <.& mg?, P 1@ mg, bolus
11 J Autonomic *harmacology= Sympathetic !ervous System
1.1 #he sympathetic nervous system $S!S and parasympathetic nervous system are
divisions of which of the following?
a Somatic nervous system division of peripheral nervous system
b Somatic nervous system division of central nervous system
c Autonomic nervous system division of peripheral nervous system
d Autonomic nervous system division of central nervous system
1.3 *reganglionic sympathetic and parasympathetic fibers release LLLL, postganglionic
parasympathetic fibers release LLLL $for muscarinic receptors, and
postganglionic sympathetic fibers release LLLL $for adrenergic receptors.
a A>hE A>hE !1
b A>hE !1E A>h
c !1E A>hE !1
d !1E !1E A>h
1.% -hich of the following adrenergic receptors is most commonly found pre-synaptic?
a Alpha 1 receptors
b Alpha 3 receptors
c !eta ( receptors
d !eta ) receptors
e !eta * receptors
1.4 -hich of the following describes the result of adrenal medulla stimulation?
a Bass parasympathetic discharge, @&=1& ratio of epi=norepi
b Bass parasympathetic discharge, 1&=@& ratio of epi=norepi
c Bass sympathetic discharge, @&=1& ratio of epi=norepi
d Bass sympathetic discharge, 1&=@& ratio of epi=norepi
Batch the sympathetic response with the receptor=
1.& Increased lipid brea.down a Y1
1.9 *eripheral vasoconstriction b Z1
1.< Increased heart rate and blood pressure c Z3
1.@ 7ronchial dilation, coronary dilation, glucose conversion d Z%
1.A -hat amino acids is converted into catecholamines $!1, 1pi, Dopamine?
a Alanine
b *roline
c ,ysine
d #yrosine
e (aline
1.15 -hich of the following is transported into vesicles via the vesicular monoamine
transporter $(BA#, upta.e 3, a proton antiporter?
a 1pinephrine
b !orepinephrine
c Dopamine
1.11 -hich of the following is co-stored and co-released with A#*?
a 1pinephrine
b !orepinephrine
c Dopamine
1.13 -hich of the following form varicosities or en passant synapses, with the arrival of
an action potential leading to >aOO influ+ and e+ocytosis?
a *resynaptic sympathetic
b *resynaptic parasympathetic
c *ostsynaptic sympathetic
d *ostsynaptic parasympathetic
3.1 -hich of the following methods of terminating a+on response is !)# a target for
drug action?
a "eupta.e via !1 transporter $!1#= Hpta.e 1
b Betabolism of !1 of inactive metabolite
c !1 diffusion away from synaptic cleft
3.3 !1# is a symporter of what ion?
a IO
b >aOO
c >l-
d !aO
e BgOO
3.% -hich of the following is recycled via (BA# into vesicles after response
termination?
a !1
b ,-D)*A
c !1#
d 1*I
e D)*CA,
3.4 -hich of the following is bro.en down by BA)-7 $monoamine o+idase more than
the others?
a Serotonin $&-'#
b !orepinepherine $!1
c Dopamine $DA
3.& -here is the cytosolic catecholamine metaboli2ing en2yme catechol-)-methyl
transferase $>)B# primarily found?
a ,iver
b CI tract
c *lacenta
d 7lood platelets
%.1 -hich of the following receptor subtypes rela+es smooth muscle and causes liver
glycogenolysis and gluconeogenesis?
a Y1 $C/?Ci?Co
b Y3 $Ci?Co
c Z1 $Cs
d Z3 $Cs
e Z% $Cs
%.3 -hich of the following receptor subtypes causes vascular smooth muscle contraction
and genitourinary smooth muscle contraction?
a Y1 $C/?Ci?Co
b Y3 $Ci?Co
c Z1 $Cs
d Z3 $Cs
e Z% $Cs
%.% -hich of the following receptor subtypes increases cardiac chronotropy $rate and
inotropy $contractility, increases A(-node conduction velocity, and increases rennin
secretion in renal Fu+taglomerular cells?
a Y1 $C/?Ci?Co
b Y3 $Ci?Co
c Z1 $Cs
d Z3 $Cs
e Z% $Cs
%.4 -hich of the following receptor subtypes decreases insulin secretion from pancreatic
Z-cells, decreases nerve cell norepinephrine release, and contracts vascular smooth
muscle?
a Y1 $C/?Ci?Co
b Y3 $Ci?Co
c Z1 $Cs
d Z3 $Cs
e Z% $Cs
4.1 -hat type$s of second messenger$s interact with adenylyl cyclase?
a Y1
b Y3
c Z
d Z G Y1
e Z G Y3
4.3 -hat type$s of second messenger$s are associated with phospholipase > $*,>?
a Y1
b Y3
c Z
d Z G Y1
e Z G Y3
4.% -hich of the following adrenergic receptor activation mechanisms is involved with
ephedrine, amphetamine, and tyramine?
a Direct binding to the receptor
b *romoting release of norepinephrine
c Inhibiting reupta.e of norepinephrine
d Inhibiting inactivation of norepinephrine
4.4 -hich of the following adrenergic receptor activation mechanisms is involved with
BA) inhibitors?
a Direct binding to the receptor
b *romoting release of norepinephrine
c Inhibiting reupta.e of norepinephrine
d Inhibiting inactivation of norepinephrine
4.& -hich of the following adrenergic receptor activation mechanisms is involved with
tricyclic antidepressants and cocaine?
a Direct binding to the receptor
b *romoting release of norepinephrine
c Inhibiting reupta.e of norepinephrine
d Inhibiting inactivation of norepinephrine
4.9 -hich of the following is !)# true of catecholamines?
a !on-polar
b >annot cross the blood-brain barrier
c >annot be used as an oral drug
d 'ave brief duration
e BA) and >)B# act rapidly
Batch the catecholamine with the receptor$s=
4.< Isoproterenol a Y G Z
4.@ Dobutamine b Z
4.A !orepinepherine c Z1
4.15 Dopamine d D1 G D3
4.11 1pinepherine
4.13 #he basic structure of a catecholamine involves a catechol ring and which of the
following types of amines?
a Bethyl amine
b 1thyl amine
c 7utyl amine
d #ert-butyl amine
e *ropyl amine
Batch the noncatecholamines with the receptor agonist=
4.1% >lonidine a Y1-agonist
4.14 Betaproterenol, terbutaline, ritodine b Y3-agonist
4.1& *henylephrine c Z3-agonist
&.1 -hich of the following is a long-acting $oral Y1-agonist and not a short-acting
$nasal spray, ophthalmic drops Y1-agonist?
a *henylephrine
b )+ymeta2oline
c #etrahydra2aline
d *seudoephedrine
&.3 -hich of the following would !)# be used as a topical vasoconstrictor for a patient
with epista+is $nasal pac. soa.ed in drug?
a *henylephrine
b 1pinepherine
c )ymeta2oline
d Isoproterenol
&.% Y1 drugs can be given with local anesthetics to vasoconstrictor and decrease blood
flow to the side of administration. -hich of the following should not be given above the
web space?
a *henylephrine
b 1pinephrine
c Betho+amine
&.4 -hich of the following is the Y1 drug of choice $D)> for retinal e+ams and
surgery, giving mydiasis $dilation of iris?
a 1phedrine
b 1pinepherine
c )ymeta2oline
d Isoproterenol
e *henylephrine
&.& Y3-agonists are only approved for hypertension and wor. by decreasing sympathetic
tone and increasing vagal tone. -hich of the following is !)# a Y3-agonist?
a >lonidine
b Bethyldopa
c Cuanaben2
d Cuanfacine
e 1pinephrine
&.9 At the adrenergic synapse, what does Y3 do?
a Stimulates !1 release
b Inhibits !1 release
c Stimulates A>h release
d Inhibits A>h release
&.< -hich of the following agonists would be used for asthma patients or to delay
premature labor?
a Y3-agonist
b Y1-agonist
c Z%-agonist
d Z3-agonist
e Z1-agonist
&.@ -hich of the following agonists would be used for cardiogenic shoc., cardiac arrest,
heart bloc., or heart failure?
a Y1-agonist
b Y3-agonist
c Z1-agonist
d Z3-agonist
e Z%-agonist
&.A -hich of the following is !)# a Z3-agonist?
a #erbutaline
b "itodrine
c Betaproterenol
d Albuterol
e *henylepherine
&.15 Z3 stimulation leads to an increase in the cellular upta.e of what ion, and thus a
decrease in plasma concentration of that ion?
a IO
b >aOO
c >l-
d !aO
e BgOO
&.11 Dopamine receptor activation $D1 dilates renal blood vessels at low dose. At
higher doses $treatment for shoc., which of the following receptor is activated?
a Y1
b Y3
c Z1
d Z3
e Z%
&.13 -hich of the following responses to sympathetic stimulation would prevent
receptors from being couples with +,proteins?
a Se/uestration
b Down-regulation
c *hosphorylation
&.1% -hich of the following is the action of the indirect-acting sympathomimetic drug
cocaine?
a Stimulator of !1# $upta.e 1
b Inhibitor of !1# $upta.e 1
c Stimulator of (BA# $upta.e 3
d Inhibitor of (BA# $upta.e 3
&.14 #ricyclic antidepressants $#>As have a great deal of side effects. -hich of the
following is the action of #>As?
a Stimulator of !1# $upta.e 1
b Inhibitor of !1# $upta.e 1
c Stimulator of (BA# $upta.e 3
d Inhibitor of (BA# $upta.e 3
&.1& -hich of the following is !)# a mi+ed sympathomimetic?
a Amphetamine
b -ethamphetamine
c 1phedrine
d *henylepherine
e *seudoephedrine
&.19 *rior to an operation to remove a pheochromocytoma $neuroendocrine tumor of the
medulla of the adrenal glands, which of the following should be given to the patient?
a Y-agonist
b Y-bloc.er
c Z-agonist
d Z-bloc.er
&.1< -hich of the following is !)# an indication for Z-bloc.er therapy?
a 'ypotension
b Angina pectoris
c Arrhythmias
d Byocardial infarction
e Claucoma
&.1@ -hich of the following Z-bloc.ers is used for decreasing a/ueous humor secretions
from the ciliary body?
a *ropranolol
b !adolol
c >arvedilol
d #imolol
e Betoprolol
&.1A -hich of the following is !)# considered cardioselective?
a Betoprolol
b Atenolol
c 1smolol
d >arvedilol
&.35 7loc.ing Y3 presynaptic receptors will do which of the following?
a Stimulate !1 release
b Inhibit !1 release
c Stimulate DA release
d Inhibit DA release
&.31 -hich of the following drugs irreversibly damages (BA#?
a #yramine
b Cuanethidine
c "eserpine
d *ropranolol
e 1pinepherine
9.1 -hich of the following is the most li.ely to occur with parenteral administration of a
Y1-agonist drug?
a 'ypotension
b 'ypertension
c #issue necrosis
d (asodilation
e ,ipolysis
9.3 -hich of the following agonists can have dose-related withdrawal syndrome if the
drug is withdrawn too /uic.ly, leading to rebound hypertension?
a Y1-agonist
b Y3-agonist
c Z1-agonist
d Z3-agonist
e Z%-agonist
9.% -hich of the following agonists can have sedation and +erostomia $dry mouth in
&5M of patients starting therapy, se+ual dysfunction in males, nauseas, di22iness, and
sleep disturbances?
a Y1-agonist
b Y3-agonist
c Z1-agonist
d Z3-agonist
e Z%-agonist
9.4 -hich of the following agonists can cause hyperglycemia in diabetics?
a Y3-agonist
b Y1-agonist
c Z%-agonist
d Z3-agonist
e Z1-agonist
9.& Angina pectoris, tachycardia, and arrhythmias are possible adverse effects of which
of the following agonists?
a Y3-agonist
b Y1-agonist
c Z%-agonist
d Z3-agonist
e Z1-agonist
9.9 If a patient is ta.ing BA) inhibitors and ingests tyramine $red wine, aged cheese,
which of the following acute responses is most li.ely? $sympathomimetic
a Stimulation of !1 release
b Inhibition of !1 release
c Stimulation of A>h release
d Inhibition of A>h release
e !o response due to BA) inhibitor
9.< -hich of the following occurs acutely, leading to a false neurotransmitter, with
increased guanethidine? $sympathomimetic
a Stimulation of !1 release
b Inhibition of !1 release
c Stimulation of A>h release
d Inhibition of A>h release
9.@ BaFor adverse affects of the Y1 bloc.ade include refle+ tachycardia and which of the
following?
a )rthostatic tachycardia
b )rthostatic bradycardia
c )rthostatic hypertension
d )rthostatic hypotension
e Increased cardiac output
9.A -hich of the following effects would be intensified with the Y3 bloc.ade?
a "efle+ tachycardia
b "efle+ bradycardia
c )rthostatic hypertension
d )rthostatic hypotension
e *latelet clotting
9.15 -hich of the following is !)# an adverse affect of the Z1 bloc.ade?
a 7radycardia
b Decreased cardiac output
c A( node bloc.
d Increased arrhythmias
e 'eart failure
9.11 -hich of the following is the most severe adverse effect that has been associated
with sudden termination of Z1-bloc.ers?
a Atrial fibrillation
b "efle+ bradycardia
c Syncope $fainting
d Angina
e Sudden death
9.13 -hich of the following groups of patients is most at ris. for adverse effect seen in
Z3-bloc.ers?
a Asthmatics
b >ongestive heart failure patients
c #rauma patients
d Diabetics
e *atients with deep vein thromboses $D(#s
9.1% -hich of the following can be detrimental in diabetics and also can lead to mas.ing
of tachycardia, which is indicative of hypoglycemia?
a Y1-bloc.er
b Y3-bloc.er
c Z1-bloc.er
d Z3-bloc.er
e Z%-bloc.er
Item !umber= 19@4 correct answer= & category= Analgesics
1. A child has ingested an un.nown substance and has evidence of
respiratory depression. #his symptom is usually found with poisoning
due to=
1. amphetamines
3. atropine
%. mushrooms
4. .erosene
&. opioids
Item !umber= 3&%4 correct answer= % category= Analgesics
3. #he use of methadone in the treatment of heroin addiction continues
to be controversial. It would therefore be advantageous to find a substance
with the beneficial effects of methadone, but without its undesirable
characteristics. #he correct statement is that=
1. methadone is not physically addicting and therefore very useful in
treating heroin addicts
3. the withdrawal syndrome of methadone is of shorter duration than that of
heroin
%. propo+yphene may successfully suppress the withdrawal syndrome in
heroin-addicted individuals
4. although propo+yphene does bloc. heroin withdrawal, it itself is not
physically addicting
&. propo+yphene may be successfully substituted for heroin in the addicted
individual and abruptly discontinued after three to four wee.s without
signs of an abstinence syndrome
Item !umber= 3<AA correct answer= % category= Analgesics
%. >hronic renal damage resulting from the ingestion of analgesics has
been suggested. #he 6A,S1 statement is=
1. phenacetin has been implicated
3. salicylates have been implicated
%. meperidine has been implicated
4. acetaminophen is a metabolite of phenacetin
&. combinations may be more harmful
Item !umber= %495 correct answer= 4 category= Analgesics
4. A 35-month old infant is brought to the emergency room with fever,
vomiting, stupor, and hyperpnea of 13 hours: duration. 'is leu.ocyte count
is 9,555?cu mm. #he chest roentgenogram is clear. Hrinalysis shows albuminuria,
a positive test for reducing substance, and acetonuria. 6erric chloride added
to the boiled acidified urine shows a persistent purple color. #he most li.ely
diagnosis is=
1. acute glomerulonephritis
3. diabetic acidosis
%. acute bacterial meningitis
4. salicylate poisoning
&. phenothia2ine poisoning
Item !umber= %@<5 correct answer= & category= Analgesics
&. #he gastric mucosa has the important ability to prevent movement of gastric
acid from the stomach lumen into the gastric wall. Some diseases and drug
regiments have been implicated as causes of increased gastric mucosal
permeability to hydrogen ion. Drugs which may increase gastric wall permeability
include=
1. erythromycin
3. indomethacin
%. nitrofurantoin
4. aspirin
&. 3,4
Item !umber= %@<1 correct answer= 4 category= Analgesics
9. In normal patients, the so-called gastric mucosa barrier protects the
mucosal lining from bac. diffusion of hydrogen ions and subse/uent destruction.
In some patients even small changes in the gastric barrier allows bac. diffusion
of significant hydrogen ion and subse/uent destructive action. -hich of the
following pharmacologic agents have been implicated as causes of gastric
barrier brea.down?
1. caffeine
3. ethanol
%. aspirin
4. 3,%
&. All of the above
Item !umber= %A5< correct answer= & category= Analgesics
<. *atients with normal platelet counts and normal bleeding time may
still bleed severely as a result of aspirin ingestion prior to a dental or
surgical procedure. #he aspirin interference with normal platelet function may
last as long as=
1. 4 hours
3. 13 hours
%. 3 days
4. & days
&. < days
Item !umber= 45AA correct answer= % category= Analgesics
@. #he appropriate antidote in the treatment of penta2ocine overdosage is=
1. nalorphine
3. levallorphan
%. nalo+one
4. Any of the above
&. !one of the above
Item !umber= 4&@< correct answer= 4 category= Analgesics
A. Acute hemorrhagic gastritis is one of the most fre/uent causes of severe
upper gastrointestinal bleeding. It is fre/uently related to recent ingestion of
ethanol and?or aspirin and may be life-threatening. 7leeding secondary to aspirin
is mainly due to=
1. inhibition of gastric prostaglandin synthesis
3. decreased renal e+cretion of the salicylate with attendant longer half-
life in serum
%. bac. diffusion of hydrogen ions across the gastric mucosa
4. 1,%
&. !one of the above
Item !umber= 49%A correct answer= % category= Analgesics
15. Bany stimuli may cause vomiting. #he chemoreceptor trigger 2one of the
central nervous system is=
1. located in the cerebral corte+
3. stimulated in all forms of vomiting
%. stimulated by morphine and its congeners
4. All of the above
&. 1,%
Item !umber= &3<& correct answer= 1 category= Analgesics
11. A pharmacologic agent which has the potential to cause increased
biliary tree pressure is=
1. morphine
3. warfarin
%. phenytoin
4. aceta2olamide
&. carbon tetrachloride
Item !umber= 9395 correct answer= 4 category= Analgesics
13. Bany commonly utili2ed medications are subFect to abuse. !ormally nonto+ic
medications, if ingested in e+cessive amounts, may cause end-organ damage.
Analgesic nephropathy has been associated with=
1. prolonged abuse of phenacetin
3. prolonged abuse of phenacetin-aspirin combinations
%. acetaminophen derived from phenacetin
4. All of the above
&. 1,%
Item !umber= 9<9& correct answer= 4 category= Analgesics
1%. All of the following statements regarding acetaminophen to+icity are
true 1K>1*#=
1. acetaminophen is the principal metabolic product of phenacetin
3. an overdose of 15 grams or more may produce hepatic necrosis in adults
%. acetaminophen overdose may produce transient a2otemia or renal failure
4. forced diuresis may be a useful form of therapy for acetaminophen overdose
&. liver pathology is centrilobular and mid2onal necrosis
Item !umber= A<<4 correct answer= 1 category= Analgesics
14. )pioid analgesics are sometimes associated with the production of
pulmonary disease. #he most common pulmonary complication after oral ingestion is=
1. pulmonary edema
3. interstitial fibrosis
%. pulmonary calcification
4. bronchoconstriction
&. pleural effusion
Item !umber= 9<5< correct answer= 1 category= Analgesics
1&. >haracteristically observed in individuals following acute overdose of
opioids.
A. pinpoint pupils
7. depressed respiration
>. coma
D. convulsions
1. A,7,>
3. A,>
%. 7,D
4. D only
&. All of the above
Item !umber= 9<5@ correct answer= 3 category= Analgesics
19. #"H1 statement concerning the actions of opioids on the secretion of pituitary
hormones.
A. suppress the secretion of luteini2ing hormone and thyrotropin
7. reduce the release of prolactin
>. act as a stimulus for AD' secretion
D. inhibit the secretion of A>#'
1. A,7,>
3. A,>
%. 7,D
4. D only
&. All of the above
Item !umber= 9<3@ correct answer= 4 category= Analgesics
1<. #he duration of analgesia is one important characteristic which differentiates
one opioid from another. -hich opioid possesses the shortest duration of
analgesia?
1. morphine
3. hydromorphone
%. codeine
4. meperidine
&. methadone
Item !umber= 9<%% correct answer= & category= Analgesics
1@. -hich of the following possess$es some antagonist activity at opioid
receptors?
A. nalo+one
7. penta2ocine
>. butorphanol
D. nalorphine
1. A,7,>
3. A,>
%. 7,D
4. D only
&. All of the above
Item !umber= 9<45 correct answer= 1 category= Analgesics
1A. #"H1 statements concerning the pharmacologic properties of salicylates
include=
A. high dose aspirin therapy can lower the serum urate concentration
7. aspirin is metaboli2ed by a combination of 1st and 2ero order
.inetics
>. aspirin e+erts its actions primarily by inhibition of
cycloo+ygenase
D. aspirin overdose causes significant hepatic to+icity if ingested
in sufficient /uantities
1. A,7,>
3. A,>
%. 7,D
4. D only
&. All of the above
Item !umber= 9<&4 correct answer= 3 category= Analgesics
35. #he opioid e+pected to have the shortest duration of action following
subcutaneous administration of e/uianalgesic doses=
1. hydromorphone
3. meperidine
%. methadone
4. morphine
&. codeine
Item !umber= 9<&9 correct answer= 3 category= Analgesics
31. A patient you follow in clinic has a well-.nown heroin abuse problem. Drugs
which could potentially prevent an abstinence withdrawal syndrome during
hospitali2ation include=
A. morphine
7. nalbuphine
>. methadone
D. butorphanol
1. A,7,>
3. A,>
%. 7,D
4. D only
&. All of the above
Item !umber= 9<95 correct answer= 1 category= Analgesics
33. #"H1 statement concerning the pharmacologic effects of salicylates include=
A. salicylates are thought to e+ert their activity at least partially
by inhibiting prostaglandin synthetase
7. high-dose salicylate therapy $T & grams?day lowers the serum uric
acid concentration
>. the effect of salicylates upon platelet aggregation is
irreversible unli.e that of other nonsteroidal anti-inflammatory
drugs
D. salicylate overdose is potentially fatalE however, prompt
administration of acetylcysteine will avert this danger
1. A,7,>
3. A,>
%. 7,D
4. D only
&. All of the above
Item !umber= 9AA5 correct answer= & category= Analgesics
3%. #he analgesic most apt to produce dysphoria=
1. morphine
3. meperidine
%. methadone
4. codeine
&. penta2ocine
Item !umber= 9AA@ correct answer= 1 category= Analgesics
34. Borphine:s affects the eye by=
1. producing miosis through an action on the oculomotor nerve
3. producing mydriasis through an action on the sympathetic system
%. decreasing pupillary responses to light
4. directly acting on the smooth muscles of the iris
&. directly acting on e+trinsic muscles of the eye
Item !umber= <554 correct answer= % category= Analgesics
3&. Aspirin is a nonopioid analgesic which is thought to wor. by inhibiting=
A. prostaglandin reductase
7. prostaglandin synthetase
>. thrombo+ane synthetase
D. cycloo+ygenase
1. A,7,>
3. A,>
%. 7,D
4. D only
&. All of the above
Item !umber= <559 correct answer= % category= Analgesics
39. Aspirin may be fatal if ta.en in sufficient /uantity. #he syndrome of acute
salicylate overdose in children is characteri2ed by=
A. mar.ed hypothermia secondary to an antipyretic effect
7. fever
>. peripheral edema
D. disturbance in acid-base and electrolyte balance
1. A,7,>
3. A,>
%. 7,D
4. D only
&. All of the above
Item !umber= 1131< correct answer= % category= Analgesics
3<. Acetaminophen has been used as a safe and effective analgesic? antipyretic
agent for over @5 years. Since it may be purchase without a prescription, it
is readily available and as such the recommended dose may be e+ceeded. #he
primary to+icity leading to death from an acetaminophen overdose is=
1. papillary necrosis and chronic interstitial nephritis
3. pancytopenia
%. hepatocellular necrosis
4. myocarditis
&. hemolytic anemia
Item !umber= 1194& correct answer= 4 category= Analgesics
3@. #he pharmacologic effects of morphine include all 1K>1*#=
1. behavioral changes
3. miosis
%. respiratory depression
4. diarrhea
&. postural hypotension
Item !umber= correct answer= 3 category= nesthetics
1. All of the following factors influence the rate of induction of anesthesia
with an inhaled anesthetic 1K>1*#=
1. a/ueous solubility of the anesthetic
3. patient history of malignant hyperthermia
%. ventilation rate
4. tension of gas administration
&. pulmonary blood flow rate
6or /uestions 3 J A, choose the correct answer from the following list. 1ach
answer may be used once, more than once, or not at all.
Anesthetic *artition Binimum Alveolar
>oefficient >oncentration $BA>
1. halothane 3.%5 5.<&
3. enflurane 1.@5 1.9@
%. nitrous o+ide 5.4< 15&.55
4. isoflurane 1.45 1.1&
&. metho+yflurane 13.55 5.19
Item !umber= correct answer= % category= Anesthetics
3. #he blood=gas partition coefficient is the ratio of anesthetic concentration
in blood compared to gas phase. Creater solubility in blood results in slower
onset of anesthesia and higher partition coefficient value. -hich anesthetic
has the fastest onset of action?
Item !umber= correct answer= & category= Anesthetics
%. #he minimum alveolar concentration $BA> is the concentration of anesthetic at
1 atmosphere of pressure that produces immobility in &5M of patients e+posed
to a no+ious stimulus. -hich anesthetic is most potent by this measure?
Item !umber= correct answer= % category= Anesthetics
4. A good analgesic at subanesthetic doses
Item !umber= correct answer= % category= Anesthetics
&. #he anesthetic that may be most safely used in a patient with a history of
malignant hyperthermia?
Item !umber= correct answer= 1 category= Anesthetics
9. Anesthetic associated with the highest incidence of hepatitis.
Item !umber= correct answer= 3 category= Anesthetics
<. #he anesthetic which should be avoided in patients with a sei2ure disorder
because tonic-clonic sei2ures are associated with its use.
Item !umber= correct answer= 4 category= Anesthetics
@. #he halogenated anesthetic which is advantageous in patients with
cardiovascular disease because it maintains cardiac output, produces systemic
and coronary vasodilation, and catecholamine dependent arrhythmias are
uncommon.
Item !umber= correct answer= & category= Anesthetics
A. #his anesthetic has become obsolete due to its potential for causing
nephroto+icity.
6or /uestions 15 J 1%, choose the correct answer from the following list. 1ach
answer may be used once, more than once, or not at all.
1. thiopental
3. propofol
%. .etamine
4. etomidate
&. fentanyl
Item !umber= correct answer= 4 category= Anesthetics
15. #he intravenous anesthetic with e+citatory effects on the central nervous
system but produces the least cardiovascular disturbance among intravenous
anesthetics.
Item !umber= correct answer= 3 category= Anesthetics
11. Anesthetic that must be administered in a lipid diluent due to poor a/ueous
solubility.
Item !umber= correct answer= % category= Anesthetics
13. In contrast to most anesthetics, this agent produces cardiac stimulation,
resulting in increased blood pressure, heart rate and cardiac output.
Item !umber= correct answer= & category= Anesthetics
1%. Anesthetic fre/uently used for analgesia during surgery. ,arge doses induce or
maintain anesthesia but mechanical ventilation re/uired due to profound
central respiratory depression.
Item !umber= correct answer= 4 category= Anesthetics
14. #rue statement regarding thiopental=
1. causes increased 11C activity
3. contraindicated in a patient with epilepsy
%. provides analgesia
4. associated with dose-dependent respiratory depression
&. provides ade/uate s.eletal muscle rela+ation alone
Item !umber= correct answer= 3 category= Anesthetics
1&. 6alse statement regarding propofol as an anesthetic=
1. pharmaco.inetic half life does !)# correlate with duration of central
nervous system depression
3. provides ade/uate s.eletal muscle rela+ation alone
%. vasodilation may produce a decline in blood pressure. >an be minimi2ed
by decreasing dose.
4. safe alternative for patients predisposed to malignant hyperthermia
&. high lipophilicity provides rapid and smooth onset, dose titratability,
and rapid recovery from anesthesia
Item !umber= correct answer= 1 category= Anesthetics
19. 6A,S1 statement regarding .etamine as an anesthetic=
1. contraindicated in a patient with asthma because it may cause
bronchoconstriction
3. recovery from anesthesia associated with emergence phenomenon of
hallucinations and vivid dreams
%. produces dissociative anesthetic state, whereby patient appears awa.e
but is unconscious
4. produces analgesia
Item !umber= correct answer= 4 category= Anesthetics
1<. >orrect statements regarding etomidate include all 1K>1*#=
1. inhibits adrenocortical function
3. no adverse effects from adrenocortical inhibition during short-term use
%. produces minimal cardiovascular effects
4. potent hypnotic with e+cellent analgesic properties
&. safe alternative for a patient with unstable cardiovascular status
Item !umber= correct answer= % category= Anesthetics
1@. #"H1 statement regarding mida2olam as an adFunct to anesthesia=
1. provides analgesia
3. provides ade/uate s.eletal muscle rela+ation alone
%. useful for ameliorating sei2ure activity
4. produces cardiovascular stimulation
&. contraindicated in patients with asthma or >)*D
Item !umber= correct answer= 3 category= Anesthetics
1A. 6A,S1 statement regarding bupivacaine, a local anesthetic with a molecular
structure that includes an amide lin.age.
1. undergoes hepatic metabolism
3. rapidly hydroly2ed by plasma esterase en2ymes
%. contraindicated in a patient with a history of allergy to lidocaine
4. higher dose necessary to induce epidural anesthesia than spinal
anesthesia
&. higher ris. of systemic side effects when used for epidural anesthesia
compared to spinal anesthesia
Item !umber= correct answer= 4 category= Anesthetics
35. 6A,S1 statement regarding local anesthetics=
1. produce effects by inhibiting voltage gated sodium channels
3. inhibition of sodium channels enhanced by repetitive depolari2ations
%. small nerve fibers more susceptible than large nerve fibers
4. all sensory functions of a nerve affected e/ually at onset of anesthesia
&. vasoconstriction prolongs duration of action
Item !umber= correct answer= & category= Anesthetics
31. #reatment of malignant hyperthermia includes all of the following 1K>1*#=
1. cessation of anesthetic and changing of rubber tubing to anesthesia
machine
3. administration of dantrolene sodium 1mg?.g by rapid intravenous infusion
until symptoms subside
%. hyperventilation of patient with 155M o+ygen
4. administration of fluids and diuretics to control myoglobinemia and
hyper.alemia
&. lidocaine is indicated for arrhythmias
Item Number/ 01)* correct ans2er/ 3 category/ Inotropes4'asodilators
(5 6igitalis glycosides slo2s the heart rate in patients 2ith systolic
dysfunction as a result of all of the follo2ing E78EPT/
(5 direct action of digitalis on the ' node to slo2 conduction
)5 indirect e9ect of enhanced vagal tone on ' node
*5 enhanced parasympathetic out:o2 from the 8NS through an indirect
mechanism
;5 diminished sympathetic tone 2hich is high as a compensatory mechanism in
heart failure
35 enhanced responsiveness of the S and ' node to norepinephrine through
an indirect mechanism
Item Number/ ((*31 correct ans2er/ ( category/ Inotropes4'asodilators
)5 <hich of the follo2ing drug,mechanism of action pairs is properly matched=
(5 digo&in > inhibits Na? > @? TPase pump
)5 dobutamine > inhibits troponin I
*5 amrinone > inhibits troponin I
;5 dopamine > inhibits troponin I
35 isoproterenol > inhibits ( and ) receptors
Item Number/ ((1( correct ans2er/ ) category/ Inotropes4'asodilators
*5 The rapid heart rate sometimes seen after nitroglycerin administration is best
e&plained by/
(5 a direct positive chronotropic e9ect on the myocardium
)5 re:e& sympathetic discharge due to a fall in systemic blood pressure
*5 the ability of nitroglycerin to release norepinephrine from sympathetic
nerve endings
;5 a decrease in intracranial pressure
Item Number/ ((3A correct ans2er/ * category/ Inotropes4'asodilators
;5 The e9ectiveness of digitalis in the treatment of atrial :utter is primarily
due to its ability to/
(5 slo2 the rate of Bring of the S, node
)5 e&ert an atropine,li.e e9ect on the ,' node
*5 slo2 conduction through the ,' node
;5 decrease the refractory period through the ,' node
35 decrease the rate of conduction through atrial muscle
Item Number/ *A)A correct ans2er/ 3 category/ Inotropes4'asodilators
35 <hen digitalis therapy is initiatedC serious cardiac arrhythmias may be caused
by a deBciency of/
(5 Na?
)5 @?
*5 8l,
;5 -g??
35 )C;
Item Number/ *01( correct ans2er/ * category/ Inotropes4'asodilators
15 The cardiac glycosides are used in the treatment of congestive heart failure
and atrial Bbrillation5 +iven the di9erences bet2een the cardiac glycosides
in their length of action and organs of maDor e&cretionC the correct statement
concerning the use of digito&in is/
(5 gastrointestinal absorption of digito&in is incomplete and highly
variable 2ithin patients
)5 digito&in is e&creted primarily unchanged in the .idney 2ith a half life
of about ); hours
*5 digito&in is highly protein bound in the serum
;5 digito&in has a much higher incidence of to&icity in patients 2ith renal
disease
35 digito&in is only available as an oral preparation
Item Number/ ((*A; correct ans2er/ ; category/ Inotropes4'asodilators
E5 6igitalis to&icity manifested by premature ventricular contractions maybe
treated 2ith all of the follo2ing E78EPT/
(5 lidocaine
)5 digitalis,speciBc immune F! antibody
*5 phenytoin
;5 "uinidine
35 ll are correct
Item Number/ ((*30 correct ans2er/ ) category/ Inotropes4'asodilators
A5 The correct statement regarding digitalis is/
(5 in normal individualsC digitalis increases cardiac output
)5 in normal individualsC digitalis increases myocardial o&ygen consumption
*5 in normal individualsC digitalis reduced total peripheral resistance in
response to enhanced myocardial contractility
;5 in normal individualsC digitalis depresses myocardial contractility
35 in normal individualsC digitalis increases heart rate
Item Number/ *E(A correct ans2er/ ( category/ Inotropes4'asodilators
05 routine electrocardiogram reveals a borderline delay in PR intervalC sagging
left ventricular ST segments and a shortened RT interval5 The most li.ely
e&planation 2ould be/
(5 digitalis e9ect
)5 acute sub,endocardial ischemia
*5 hypo.alemia
;5 hypocalcemia
35 my&edema
Item Number/ ((*1E correct ans2er/ ; category/ Inotropes4'asodilators
(F5 n uncommon cardiac manifestation of digitalis to&icity is/
(5 premature ventricular contractions
)5 second and third degree heart bloc.
*5 atrio,ventricular Dunctional escape beats
;5 atrial Bbrillation 2ith rapid ventricular response
35 ll are correct
Item Number/ ;E3( correct ans2er/ ) category/ Inotropes4'asodilators
((5 The most 2idely used digitalis glycosides in the Gnited States are digo&in and
digito&in5 The half,life #t(4)% of digito&in is appro&imately/
(5 ( day
)5 E days
*5 () days
;5 dependent upon the route of administration
35 dependent upon renal function
Item Number/ ((0 correct ans2er/ ( category/ Inotropes4'asodilators
()5 The maintenance dose of digo&in is primarily dependent upon/
(5 renal function
)5 se&C hepatic function and protein level
*5 pulmonary function
;5 the loading dose and the type of diuretic being used
35 level of serum potassium
Item Number/ ((;FA correct ans2er/ ; category/ Inotropes4'asodilators
(*5 Thiocyanate to&icity is a potential adverse e9ect associated 2ith/
5 amrinone
!5 nitroglycerin
85 milrinone
65 nitroprusside
(5 C!C8
)5 C8
*5 !C6
;5 6 only
35 ll are correct
Item Number/ )E)) correct ans2er/ ; category/ Inotropes4'asodilators
(;5 ;),year,old 2hite male 2ith rheumatic mitral insuHciency is seen 2ith
symptoms and Bndings of initial left ventricular failure5 Iou give him
digitalis and recall that all of the follo2ing are pharmacologic e9ects of
digitalis E78EPT/
(5 decreases conduction velocity in the ,' node
)5 increases the e9ects of normal vagal activity of the heart
*5 shortens the e9ective refractory period of the Pur.inDe Bbers
;5 2ea.ens myocardial contractility
35 prolongs the functional refractory period of the ,' node
Item Number/ ((0* correct ans2er/ * category/ Inotropes4'asodilators
(35 $rthostatic hypotension is most li.ely to occur follo2ing the use of/
(5 phenylephrine
)5 reserpine
*5 amyl nitrite
;5 metaraminol
35 tetrahydrozoline
Item Number/ ;1FF correct ans2er/ ) category/ Inotropes4'asodilators
(15 The chemical conBguration of a molecule may determine its degree of
absorptionC its distributionC and its route of metabolism5 StructurallyC the
digitalis glycosides resemble/
(5 catecholamines
)5 steroids
*5 salicylates
;5 nitrofurantoin
35 phenothiazines
Item Number/ (*10E correct ans2er/ * category/ Inotropes4'asodilators
(E5 ll of the follo2ing are actions of nitrates in angina pectoris E78EPT/
(5 re:e& increase in heart rate
)5 ventricular size decrease
*5 eDection fraction increased
;5 peripheral venous pooling>decrease in preload
35 systolic blood pressure decrease
Item Number/ ((*0A correct ans2er/ ; category/ Inotropes4'asodilators
(A5 The sympathomimetic 2hich may promote diuresis by a direct e9ect on the
.idney is/
(5 isoproterenol
)5 dobutamine
*5 norepinephrine
;5 dopamine
35 epinephrine
Item Number/ (*EF; correct ans2er/ ; category/ Inotropes4'asodilators
(05 Nitroglycerin administered sublingually reduces venous toneC causing pooling
of blood in peripheral veins and lo2ers peripheral arterial resistanceC
resulting in a decline in blood pressure5 !oth end,diastolic and end,systolic
dimensions of the left ventricle are reducedJ therefore myocardial o&ygen
needs are decreased primarily by a reduction of/
(5 blood pressure
)5 heart rate
*5 coronary vascular resistance
;5 intramyocardial tension
35 sulfhydryl bonds
Item Number/ AF0F correct ans2er/ * category/ Inotropes4'asodilators
)F5 6igitalis must be used 2ith caution in patients 2ith acute myocardial
infarction5 <hich of the follo2ing is true of digitalis use=
5 in normal heartsC it increases contractility and myocardial $)
consumption
!5 it is recommended in uncomplicated myocardial infarction
85 in failing heartsC it decreases or leaves unchanged myocardial $)
consumption
65 in acute myocardial infarctionC patients are less sensitive to the
development of arrhythmias due to digitalis to&icity
(5 only
)5 C!C8
*5 C8
;5 !C6
35 6 only
Item Number/ ((1F correct ans2er/ * category/ Inotropes4'asodilators
)(5 6igitalis may slo2 the heart rate by/
(5 bloc.ing the activity of the carotid and aortic arch baroreceptors
)5 e&erting an atropine,li.e e9ect on the ,' node
*5 stimulation of vagal center in the medulla to increase e9erent Bring
;5 increasing the rate of conduction through the ,' node
35 None of the above
Item Number/ ((;() correct ans2er/ ; category/ Inotropes4'asodilators
))5 8ompared to digito&inC digo&in is or has/
(5 more highly protein,bound
)5 more e&tensive enterohepatic circulation
*5 prolonged elimination half,life
;5 less completely absorbed orally
35 more e&tensively hepatically cleared
Item Number/ ((*0F correct ans2er/ ; category/ Inotropes4'asodilators
)*5 The beneBcial e9ects of digitalis are derived from its e9ect on myocardial
contractility and on myocardial conduction and e&citability5 In therapeutic
dosesC digitalis/
(5 slo2s the atrial rate in atrial Bbrillation
)5 enhances atrioventricular conduction
*5 slo2s the atrial rate in sinus tachycardias
;5 slo2s the ventricular rate in atrial Bbrillation
35 decreases ventricular automaticity
Item Number/ ;101 correct ans2er/ * category/ Inotropes4'asodilators
);5 6igitalis,induced emesis is/
(5 only seen 2hen digo&in is given orally
)5 of no true clinical importance
*5 due to stimulation of the chemoreceptor trigger zone
;5 related to Na? @? TPase inhibition 2ithin the +I tract
35 commonly seen 2ith rapid parenteral administration
Item Number/ ;30* correct ans2er/ ; category/ Inotropes4'asodilators
)35 6igitalis has a beneBcial e9ect on the failing myocardium via its action to
increase the availability of calcium to the contractile machinery5 s 2ith
many drugsC digitalis has also been overused5 The correct statement
concerning the use of digitalis in the ischemic and non,ischemic myocardium
is/
(5 there is a net increase in o&ygen consumption in the non,failing
myocardium
)5 there is a net increase in o&ygen consumption in the failing dilated
myocardium
*5 digitalis has proven value in prophyla&is for diastolic dysfunction
;5 digitalis causes vasoconstriction in the normal heart secondary to
enhanced sympathetic out:o2 and bloc.ing the Na?,@? TPase in smooth
muscle
35 digitalis causes an increase in heart rate in the normal heart
Item Number/ ((;)* correct ans2er/ ( category/ ntiarrhythmics
(5 Properties of the class I antiarrhythmics include
5 reduce automaticity of his,Pur.inDe Bbers
!5 decrease the rate of rise and amplitude of phase F depolarization
85 prolong P,R and K,T intervals
65 facilitate in2ard sodium conductance during rapid depolarization
(5 C!C8
)5 C8
*5 !C6
;5 6 only
35 ll are correct
Item Number/ ((;*) correct ans2er/ ; category/ ntiarrhythmics
)5 <hich of the follo2ing antiarrhythmics may 2orsen angina symptoms by
increasing myocardial o&ygen re"uirements=
5 lidocaine
!5 verapamil
85 propranolol
65 disopyramide
(5 C!C8
)5 C8
*5 !C6
;5 6 only 5
35 ll are correct
Item Number/ ((;** correct ans2er/ ; category/ ntiarrhythmics
*5 8inchonism is a dose,related adverse reaction associated 2ith 2hich of the
follo2ing antiarrhythmics=
5 tocainide
!5 procainamide
85 disopyramide
65 "uinidine
(5 C!C8
)5 C8
*5 !C6
;5 6 only
35 ll are correct
Item Number/ ((;;E correct ans2er/ ; category/ ntiarrhythmics
;5 ddition of propranolol to the drug regimen of a patient receiving lidocaine
for premature ventricular contractions after a myocardial infarction results
in/
5 increased li.elihood of brea.through P'8s
!5 displacement of protein,bound lidocaine
85 reduced renal clearance of lidocaine
65 increased li.elihood of lidocaine to&icity
(5 C!C8
)5 C8
*5 !C6
;5 6 only
35 ll are correct
Item Number/ ((;;A correct ans2er/ ( category/ ntiarrhythmics
35 Electrophysiologic properties possessed by lidocaine include/
5 reduction of e9ective refractory period in normal his,Pur.inDe
Bbers
!5 prolonged e9ective refractory period in ischemic his,Pur.inDe
Bbers
85 shortened K,T interval
65 prolonged P,R interval
(5 C!C8
)5 C8
*5 !C6
;5 6 only
35 ll are correct
Item Number/ ((;1; correct ans2er/ 3 category/ ntiarrhythmics
15 <hich of the follo2ing are classiBed as I! antiarrhythmics=
5 lidocaine
!5 phenytoin
85 tocainide
65 me&iletine
(5 C!C8
)5 C8
*5 !C6
;5 6 only
35 ll are correct
Item Number/ ((;11 correct ans2er/ ( category/ ntiarrhythmics
E5 In treating supraventricular arrhythmiasC 2hich of the follo2ing reduces
ventricular response primarily by increasing the e9ective refractory period
of the ,' node=
5 propranolol
!5 digo&in
85 verapamil
65 tocainide
(5 C!C8
)5 C8
*5 !C6
;5 6 only
35 ll are correct
Item Number/ ((;E; correct ans2er/ ( category/ ntiarrhythmics
A5 Potential side e9ects associated 2ith amiodarone therapy include/
5 pneumonitis
!5 pseudocyanosis
85 photosensitivity
65 parotiditis
(5 C!C8
)5 C8
*5 !C6
;5 6 only
35 ll are correct
Item Number/ ((;E3 correct ans2er/ * category/ ntiarrhythmics
05 8lass I8 antiarrhythmics possess 2hich of the follo2ing electrophysiologic
properties=
5 mar.edly prolong repolarization
!5 mar.edly depress phase F depolarization
85 inhibit calcium entry during plateau portion of action potential
65 inhibit sodium entry during phase F depolarization
(5 C!C8
)5 C8
*5 !C6
;5 6 only
35 ll are correct
Item Number/ ((;E1 correct ans2er/ ( category/ ntiarrhythmics
(F5 dverse e9ects associated 2ith disopyramide therapy include/
5 urinary retention
!5 constipation
85 blurred vision
65 lupus syndrome
(5 C!C8
)5 C8
*5 !C6
;5 6 only
35 ll are correct
Item Number/ )A0 correct ans2er/ 3 category/ ntiarrhythmics
((5 ntiarrhythmic drugs can be separated into four groupsC types I to I'5 ll of
the follo2ing are type I antiarrhythmic drugs E78EPT/
(5 "uinidine
)5 lidocaine
*5 procainamide
;5 phenytoin
35 propranolol
Item Number/ )0( correct ans2er/ 3 category/ ntiarrhythmics
()5 Individual antiarrhythmic agents can alter the surface electrocardiogram5
8haracteristics of "uinidine include/
(5 decreased PR interval
)5 increased KRS duration
*5 increased KT interval
;5 ll of the above
35 )C*
Item Number/ )0) correct ans2er/ 3 category/ ntiarrhythmics
(*5 Kuinidine to&icity includes all of the follo2ing E78EPT/
(5 thrombocytopenia
)5 diarrhea
*5 cinchonism
;5 ventricular Bbrillation
35 hypertension
Item Number/ )0* correct ans2er/ ( category/ ntiarrhythmics
(;5 $f the follo2ing antiarrhythmic agentsC the one 2hich most closely resembles
procainamide 2ith respect to electrophysiologic e9ects and mechanism of
action is/
(5 "uinidine
)5 lidocaine
*5 phenytoin
;5 propranolol
35 bretylium
Item Number/ (*1F correct ans2er/ 3 category/ ntiarrhythmics
(35 !ronchiolar constriction is a rare but potentially dangerous side e9ect of/
(5 "uinidine
)5 lidocaine
*5 procainamide
;5 phenytoin
35 propranolol
Item Number/ (3EA correct ans2er/ ; category/ ntiarrhythmics
(15 The e9ect of di9erent antiarrhythmic agents is best understood by .no2ing
their predominant actions on cardiac Bbers5 ll of the follo2ing general
statements are true E78EPT/
(5 "uinidine slo2s the rate of depolarization of cardiac action potentials
and increases the refractory period
)5 lidocaine and phenytoin are class ( ! antiarrhythmic
*5 beta,receptor bloc.ing drugs act by reducing the slope of the pacema.er
potential
;5 procainamide acts as a speciBc calcium antagonist
35 bretylium prolongs the action potential and so prolongs the absolute
refractory period
Item Number/ 03(A correct ans2er/ ; category/ ntiarrhythmics
(E5 In the pharmacologic treatment of cardiovascular disordersC "uinidine has all
of the follo2ing e9ects E78EPT/
(5 prolongation of the e9ective refractory period
)5 slo2s conduction velocity
*5 reduces spontaneous fre"uency of discharge of ectopic pacema.er
;5 has a positive chronotropic e9ect
35 produces peripheral vasodilation in large doses
Item Number/ 0A0* correct ans2er/ * category/ ntiarrhythmics
(A5 <hich of the follo2ing medications is contraindicated in a patient 2ith
untreated complete heart bloc.=
(5 atropine
)5 prednisone
*5 "uinidine
;5 isoproterenol
35 hydrochlorothiazide
Item Number/ ((*A1 correct ans2er/ ( category/ ntiarrhythmics
(05 The therapeutic rationale for the use of "uinidine or procainamide includes
their ability to/
(5 depress ectopic myocardial automaticity
)5 enhance myocardial membrane responsiveness
*5 decrease the e9ective refractory period
;5 (C)
35 )C*
Item Number/ ()EAA correct ans2er/ * category/ ntiarrhythmics
)F5 ll of the follo2ing are common side e9ects of "uinidine administration
E78EPT/
(5 diarrhea
)5 nausea and vomiting
*5 dry mouth
;5 tinnitus
35 headache
Item Number/ (;;3E correct ans2er/ ; category/ ntiarrhythmics
)(5 ); hours after an acute myocardial infarctionC a ;1,year,old male is being
treated 2ith a continuous intravenous drip of an antiarrhythmic drug to
suppress fre"uent multifocal premature ventricular contractions5 Le develops
generalized seizure activity5 The seizure activity can be most readily
e&plained by/
(5 ventricular tachycardia
)5 systemic embolization
*5 systemic hypotension
;5 lidocaine to&icity
35 ventricular asystole
Item Number/ correct ans2er/ ( category/ ntiarrhythmics
))5 TRGE statements concerning 'aughan,<illiamsMs 8lass III antiarrhythmic
drugs include all E78EPT/
(5 encainideC :ecainideC and propafenone are in this class
)5 bretyliumC amiodaroneC and sotalol are in this class
*5 possess diverse pharmacologic e9ects but share the capacity to prolong
action potential duration and refractoriness in Pur.inDe Bbers
;5 bloc. out:o2 of potassium during cell repolarization
35 do not alter phase F depolarization or resting membrane potential
Kuestions )* > )0
8hoose the correct ans2er for the follo2ing "uestions from the list belo25
Each ans2er can be used onceC more than onceC or not at all5
(5 bretylium
)5 amiodarone
*5 adenosine
;5 :ecainide
35 lidocaine
Item Number/ correct ans2er/ 3 category/ ntiarrhythmics
)*5 In 8LS algorithmC Brst antiarrhythmic to be administered in the treatment of
ventricular Bbrillation refractory to deBbrillation and epinephrine5
Item Number/ correct ans2er/ * category/ ntiarrhythmics
);5 6rug of choice for paro&ysmal supraventricular tachycardia unresponsive to
vagal maneuvers5
Item Number/ correct ans2er/ ( category/ ntiarrhythmics
)35 Produces transient increase in blood pressure follo2ed by hypotension due to
an initial release of norepinephrine from neurons follo2ed by bloc.ade of
norepinephrine re,upta.e5
Item Number/ correct ans2er/ ) category/ ntiarrhythmics
)15 8ontains iodine in molecular structureC is similar to thyro&ineC and use is
associated 2ith hyper, or hypothyroidism in )N > 3N of patients5
Item Number/ correct ans2er/ ) category/ ntiarrhythmics
)E5 Pharmaco.inetic properties characterized by very high protein,binding #0050N%C
very large volume of distribution #11 L4.g%C and average elimination half,life
of )3 days5
Item Number/ correct ans2er/ ; category/ ntiarrhythmics
)A5 class I8 antiarrhythmicC characterized by high aHnity for and slo2
dissociation from fast sodium channels
Item Number/ correct ans2er/ 3 category/ ntiarrhythmics
)05 class I! antiarrhythmicC characterized by rapid association > dissociation
from sodium channels
Item Number/ correct ans2er/ ; category/ ntiarrhythmics
*F5 8orrect statements concerning propafenone include all E78EPT/
(5 class I8 antiarrhythmic
)5 slo2s actions potential conduction by bloc.ing sodium channels
*5 possesses 2ea. beta bloc.ing activity
;5 class III antiarrhythmic
35 increases duration of KRS comple&
Item Number/ correct ans2er/ ( category/ ntiarrhythmics
*(5 8orrect statements concerning sotalol include all E78EPT/
(5 prolongs action potential duration and e9ective refractory period by
bloc.ing sodium channels5
)5 prolongs cell repolarization by bloc.ing potassium channels5
*5 produces bradycardia
;5 prolongs K,T interval
35 increases e9ective refractory period
Kuestions *) > *3
8hoose the correct ans2er for the follo2ing "uestions from the list belo25
Each ans2er can be used onceC more than onceC or not at all5
(5 propranolol
)5 acebutolol
*5 sotalol
;5 esmolol
35 butorphanol
Item Number/ correct ans2er/ ; category/ ntiarrhythmics
*)5 8ardioselective beta bloc.er 2ith a short elimination half,life administered
intravenously only5
Item Number/ correct ans2er/ * category/ ntiarrhythmics
**5 Nonselective beta bloc.er 2hich slo2s heart rate and also prolongs action
potential duration by bloc.ing transmembrane potassium currents5
Item Number/ correct ans2er/ ) category/ ntiarrhythmics
*;5 8ardioselective beta bloc.er possessing intrinsic sympathetic activity and
given by oral administration only5
Item Number/ correct ans2er/ ( category/ ntiarrhythmics
*35 Nonselective beta bloc.er possessing "uinidine,li.e membrane stabilizing
e9ects5
Item Number/ correct ans2er/ 3 category/ ntiarrhythmics
*15 FLSE statement concerning use of calcium channel bloc.ers as antiarrhythmics/
(5 slo2s in2ard calcium current thereby decreasing the rate of spontaneous
phase ; depolarization in Pur.inDe Bbers
)5 slo2s conduction velocity through the atrio,ventricular node and
increases functional refractory period
*5 useful for slo2ing ventricular rate in atrial Bbrillation
;5 hypotension may be a limiting side e9ect
35 verapamilC diltiazemC and nifedipine all e&ert e"ually e9ective
antiarrhythmic actions
Item Number/ correct ans2er/ * category/ ntiarrhythmics
*E5 8$RRE8T statement concerning the antiarrhythmic drug adenosine/
(5 undergoes e&tensive hepatic metabolism
)5 produces coronary vasoconstriction
*5 causes fre"uent but transient post,conversion arrhythmias
;5 increases sinus node rate
35 increases atrio,ventricular conduction
Item Number/ correct ans2er/ ( category/ ntiarrhythmics
*A5 True statement concerning adenosine include all E78EPT/
(5 administered sublingually
)5 negative chronotrope
*5 negative dromotrope
;5 negative inotrope
35 may precipitate bronchospasm
Item Number/ correct ans2er/ 3 category/ ntiarrhythmics
*05 FLSE statement concerning adenosine/
(5 half,life of ( > (F seconds
)5 higher doses necessary in patients ta.ing theophylline
*5 chest pain and shortness of breath are common but short,lived side
e9ects
;5 indicated for treating paro&ysmal supraventricular tachycardia
35 less e9ective than verapamil for treating paro&ysmal supraventricular
tachycardia
Item Number/ correct ans2er/ ) category/ ntiarrhythmics
;F5 8orrect statements about epinephrine administration for ventricular
Bbrillation include all E78EPT/
(5 ( mg I' push every * > 3 minutes
)5 F5( mg I' push every * > 3 minutes
*5 F5( mg4.g I' push every * > 3 minutes
;5 ( mg I' push follo2ed by * mg I' push follo2ed by 3 mg I' push * minutes
apart
35 )53 mg diluted to a total volume of (F ml in saline solution via an
endotracheal tube 2hen no I' access is available
Buprenorphine is a?
A. Partial mu agonist
B. Full mu agonist
C. Partial kappa agonist
D. Full kappa agonist
Which of the following is an integrase inhibitor currently in use for the treatment of HIV infection?
A. Indinavir
B. Elvitegravir
C. Saquinavir
D. Raltegravir
Item !umber= 15A9@ correct answer= 3 category= Anticoagulants
1. A patient diagnosed with a deep venous thrombosis is begun on a heparin
infusion of 1555 H?'" after a &555 H 7olus. #he earliest you can draw blood
for an a*## reflecting steady-state heparin is=
1. 4 hours
3. <.& hours
%. 13 hours
4. 34 hours
&. !one of the above
Item !umber= 15A<% correct answer= 3 category= Anticoagulants
3. #he direct-acting plasminogen activator is=
1. strepto.inase
3. uro.inase
%. epsilon-aminocaproic acid
4. anistreplase
&. heparin
Item !umber= 15A@5 correct answer= 4 category= Anticoagulants
%. #he antithrombotic drug which inhibits cycloo+ygenase en2ymes is=
1. prednisone
3. dipyridamole
%. trane+amic acid
4. aspirin
&. ticlopidine
Item !umber= 15A@% correct answer= % category= Anticoagulants
4. >orrect statements about the action of warfarin include=
A. warfarin is structurally similar to vitamin I and competitively
inhibits liver synthesis of clotting factors
7. warfarin inhibits vitamin I epo+ide reductase
>. the onset of warfarin action may be prolonged due to the long
half-life of factor (II
D. factors II, (II, IK, and K are not activated $carbo+ylated when
warfarin is administered in therapeutic doses
1. A,7,>
3. A,>
%. 7,D
4. D only
&. All of the above
Item !umber= 15A@@ correct answer= % category= Anticoagulants
&. Drugs which interact with warfarin to increase its anticoagulant effect
include=
A. ranitidine
7. trimethoprim?sulfametho+a2ole
>. rifampin
D. nonsteroidal anti-inflammatory drugs
1. A,7,>
3. A,>
%. 7,D
4. D only
&. All of the above
Item !umber= 1151@ correct answer= 1 category= Anticoagulants
9. #he mechanism of action of mini-dose heparin is correctly described by=
A. factor K is more sensitive to heparin than are other serine
protease factors
7. mini-dose heparin is useful prophylactically rather than during
active clot formation
>. by inhibiting factor K activation, a relative bloc. of the
clotting cascade develops, with less thrombin formed
D. the maFor action is to directly inhibit factor II, which is very
sensitive to heparin
1. A,7,>
3. A,>
%. 7,D
4. D only
&. All of the above
Item !umber= 11541 correct answer= 3 category= Anticoagulants
<. #he correct statement about fibrinolytic agents is=
1. strepto.inase acts directly to activate plasminogen
3. uro.inase acts directly to activate plasminogen
%. tissue-type specific plasminogen activator is more fibrin specific than
strepto.inase
4. side effects of t-*A include s.in rash, allergic reactions, and
development of antibodies
&. have been shown to increase mortality when given in the setting of an
acute myocardial infarction

You might also like